Sunday, February 3, 2013

Craig versus Rosenberg


Theist philosopher William Lane Craig debated atheist philosopher Alex Rosenberg at Purdue University on February 1.  You can watch the debate here.  I put forward my own detailed critique of Rosenberg’s book The Atheist’s Guide to Reality in a ten-part series of posts, of which you can find a roundup here.  As I’ve said before, one of Rosenberg’s strengths is that he is willing consistently to follow out the implications of scientism (however absurd and self-defeating, as we saw in the series of posts just referred to) in a way many other atheists do not.  Another is that, as this event indicates, he has (as a certain other prominent atheist famously appears not to have) the courage and intellectual honesty to debate the most formidable defenders of theism.

264 comments:

  1. A generous post Ed, respect.

    ReplyDelete
  2. 'Which is what everybody means by God.'

    I'm watching the recording and that's how Craig just ended the first argument in his opening statement. I wonder if he's been brushing up on his Aquinas.

    Man, imagine how impressive Craig's gonna be when he becomes a full-blooded Thomist. I pray to God that it won't take long.

    ReplyDelete
  3. Ed, any opinion on who you think won?

    ReplyDelete
  4. Dr Feser,

    As a Thomist, what do you make of Craig's Kalam argument. Are Thomist's committed to a B-theory of time?

    ty

    ReplyDelete
  5. I can honestly say that Rosenberg appalled me with his attitude. His opening remarks were incredibly rude. I came to the video of the debate (http://youtu.be/bhfkhq-CM84) having read several summaries, and I had figured that their summaries of Rosenberg were simply unfortunate excesses brought on by some bias. Not so! This fellow had no class whatsoever in the debate, and his responses and "arguments" were disappointing for a philosopher of his supposed stature.

    @Anonymous

    "Generous" is a generous understatement, methinks.

    ReplyDelete
  6. Rosenbegrg should have his philosophers license taken away. I lead a Ratio Christi apologetic club on campus and any student that had two years of our discussion session would have been able to beat Rosenberg.

    It was quite clear that Rosenberg had not read any new research for the last 25 years in these areas. Shame.

    Craig also came out with some new arguments for Theism just for Rosenberg. Then, for desert, he utterly, utterly destroyed Rosenberg's book. I have not seen such intellectual devastation wrought in under 10 minutes before.

    ReplyDelete
  7. 55
    Some of the more disappointing moments from Rosenberg:

    (1) the "faith is by definition belief without evidence" claim

    (2) trotting out the Euthyphro Dilemma as if it hadn't been addressed since Plato

    (3) relying on the logical POE instead of the much more tenable evidential POE

    (4) the silly "Craig uses the same arguments, which have been responded to, hence Craig doesn't listen to its critics," all the while blithely ignoring the fact that Craig has responded rather effectively to the responses (not to mention failing to explain just why Craig's arguments fail, the PSR reference excepted)

    That said, there's no shame in losing a debate to Craig. I suspect that Craig could defend atheism as well as he defends theism in a formal debate, which brings up one point of agreement on my part with Rosenberg -- I would have much rather listened to an informal discussion between Craig and Rosenberg than to a formal debate (though, as was illustrated during Craig's discussion with Kagan, it's difficult to ensure equal time in such a format, especially when you're engaging with someone as voluble as Kagan!).

    ReplyDelete
  8. I honestly expected more from rosenberg, who proved to be an embarrassment to philosophy.

    I actually like him because he is one of the few that take atheism to its logical conclusion. Unlike all the other cowards who pretend to be atheists but still like to borrow the ideals of Theism.

    Craig smashed rosenberg worse than BigFoot smashed Overeem last night in UFC 156. ;-)

    ReplyDelete
  9. SO anyone up for some Super Bowl???

    I know that KiloPapa the most worthless troll in this blog will probably watch it XD.

    ReplyDelete
  10. anonymous @ 12:59, haha!

    Rosenberg @ 43:30: 'the PSR is the principle that every that exists must have a cause' ....

    worthless.

    ReplyDelete
  11. Edward Feser (and anyone interested in responding)

    I have a serious question bout something rosenberg claims...

    He insists that the conclusions he draws in his book about what is evidently a nihilistic worldview are a result of science and not metaphysical naturalism.

    How is that to be understood exactly? All science must be interpreted via some metaphysical outlook. I don't think anyone would deny that. Also, I don't see at all how science leads to metaphysical naturalism or nihilism of any sort.

    If as he says physics fixes the facts, then how is that any different from materialistic determinism, which is of course a worldview.

    Is he trying to appeal to science to make his claims more appealing? It seems to me that he is trying to appeal to the authority of science in order to sneak in his metaphysical commitments. But isn't that a sleigh of hand?

    ReplyDelete
  12. @grodriguez (or any other physicist amongst us),

    Rosenberg kept saying that the decay of radioactive material being unpredictable (i.e. in one case an alpha particle is emitted while in another is not) means that it's an effect without a cause. He claims that all such radioactive particles are identical and since one emits/decays and the other doesn't then the decay is one without cause.

    I wanted to hear you opinion on this as a physicist if you don't mind.

    ReplyDelete
  13. After seeing Craig's opening statement, I am almost certain that Craig has been reading Feser lately. That's a good thing.

    By sheer coincidence, I was at Purdue that night, but I was there for a graduate school interview. I wanted to arrange my schedule to go, but I thought it best to focus on the real reason I was there.

    ReplyDelete
  14. Rosenberg keeps saying that neuroscience proves that there is no intentionality.

    How exactly does neuroscience do that? Anyone have an idea?

    ReplyDelete
  15. Well, either you have a way to prove that intentionality and non-intentionality have different ways to instantiate and then it will have two different types of results, and the only results plausible and ever saw up to this day are the non-intentionality ones.

    Or maybe he is concluding from the idea that if you can make science works with a non-intentional philosophy then you should conclude that intentionality doesn't exist.

    Or maybe there is a plate in everybody's brains that is written in Klingon... (This brain has no intentioality) *Personally I like this one.*

    Well I only see these 3 ways for now, to back up the claim...

    ReplyDelete
  16. Anon

    You mean Ismael, Grodrigues is the mathematician XD. Pretty close but the mathemmaticians get really mad at certain mathematical sleigh of hands physicists do XD. *Yeah internal joke U_U*

    Well Ismael may be able to clear it up to you, but I think that Rosenberg's is not correct, I mean usually decays happen because the atom is not stable or something in the quantum "world" is not stable, so it produces a particle that stabilizes the system. At least that is what I understood from the whole thing.

    Now IF, I am right maybe rosenberg is confusing something, I think either he thinks that causes are always particles which invalidates stability as a cause of something, and forces the alpha particles to be produced by other particles that in principle or in theory can be detected but are never detected in any experiment

    Or he thinks that an unstable atom is always unstable but since it doesn't automatically "tries" to stabilize itself, coupled with an idea that given the initial state A you will produce state B, then state B which in this case is the decay state, doesn't really have a cause coming from these two premises.

    I don't really see how to reach the conclusion in any other way. Unless of course I am missing something in quantum theory that open the way to Rosenberg.

    ReplyDelete
  17. I hardly agree with Rosenberg on anything, but I do agree with him when he claims that free will can in principle be refuted by science.

    It's pretty clear that we don't need to do philosophy to see that a system of billiard balls ricocheting against each other on a pool table is a system that totally lacks free will (any robust conception of free will anyway). Why can't we just extrapolate from this 2-D system of billiard balls to a 3-D system composed of neurons, cells, etc., and observe whether or not our bodily actions are completely determined?

    ReplyDelete
  18. Well you mean Libertarian Free Will.

    And does the balls really show that???

    You are getting something in the experiment and saying it is against libertarian free will, now I would bet it is either that we can predict what the balls will do OR because of some metaphysical principle related to lack of intention in the balls?

    I don't think Science by itself can answer the question, we must first define, discuss, model and predict each of the possibilities before going to the lab to get data. (see Rosenberg loves Scientism XD which doesn't work here anyway)

    ReplyDelete
  19. BTW, watch out for substance dualism, it avoids that problem XD.

    ReplyDelete
  20. Actually thinking again, it doesn't necessarily does that .... so scrap the last comment.

    ReplyDelete
  21. I have never understood why the inability of current models of quantum mechanics to predict when and where a virtual particle will appear is any more remarkable than the inability of the theory of gravity to predict when and where an apple will fall. Causation does not mean predictability.

    The same goes for free will. Eighty percent or more of human behavior is on autopilot; but if the fruits of intellection are no more than the winds of chance blowing through the branches of the neural tree, why should the whispers of Rosenberg mean anything more than the whispers of the trees in the night wind? And has anyone considered that the motion of billiard balls actually has intention? Someone struck the cue stick, no?

    ReplyDelete
  22. "Rosenberg keeps saying that neuroscience proves that there is no intentionality."

    What was his argument? Does he even know what intentionality is?

    ReplyDelete
  23. @Anonymous:

    "Rosenberg kept saying that the decay of radioactive material being unpredictable (i.e. in one case an alpha particle is emitted while in another is not) means that it's an effect without a cause. He claims that all such radioactive particles are identical and since one emits/decays and the other doesn't then the decay is one without cause."

    Craig responded by mentioning the different interpretations of QM; Rosenberg rebutted by saying that this is a matter of *experimental physics* not of QM interpretation. I think Rosenberg is incorrect and Craig's answer suffices to defuse this. In order to see why, let us go through Rosenberg's argument.

    (1) Put two lumps of radioactive material, A and B, side by side.

    (2) They are identical in all physical features.

    (3) At time t, A emits an alpha particle and B does not.

    (4) We can hear now Rosenberg ask rhetorically, what can account for the difference? Why nothing of course, because of 2 and B did not emit any alpha particle.

    (5) Ergo, the event "A emitted an alpha particle" is uncaused.

    As the TheOFloinn, indeterminacy is *not* lack of causation, but let us forego that for the sake of argument. It still does not follow what Rosenberg thinks it does. For what does (2) mean? Remember that Rosenberg insists that it is a matter of *experiment*. It is impossible, even in principle, to have two identical lumps of radioactive material so (2) is impossible to replicate in experimental practice. Maybe Rosenberg will concede and then say, forget the lumps and take just two radioactive atoms. But (2) is still impossible to obtain, even in principle, for among other things, they will be different in space-time location or in some other accident for otherwise we would not know that they are different. Heisenberg's principle says that it is impossible for us to have an experimental setup with two identical atoms even with respect to energy or other similar observables. So what can Rosenberg be appealing to? Ultimately, it can only be to quantum indeterminacy and to some metaphysical interpretation of it. Kick in Craig's response to defuse it.

    ReplyDelete
  24. Grodrigues....

    You a physicist too???

    *I haven't noticed that seriously*

    ReplyDelete
  25. To add on to Grod's reply, I'd offer the following.

    One common demand on the part of atheists is for something that defies materialist explanation. Ignoring all the problems with how that's usually put - notice that that demand suddenly goes away when talking about quantum physics.

    I agree with all of the responses Grod and TOFlinn gave, but I'd go one further. Let's look at the summarized argument again:

    (1) Put two lumps of radioactive material, A and B, side by side.

    (2) They are identical in all physical features.

    (3) At time t, A emits an alpha particle and B does not.

    (4) We can hear now Rosenberg ask rhetorically, what can account for the difference? Why nothing of course, because of 2 and B did not emit any alpha particle.

    (5) Ergo, the event "A emitted an alpha particle" is uncaused.


    Putting aside the valid criticisms, there's another problem. At 4: what can account for the difference? The answer isn't "nothing", even granting all the wrong assumptions. It would be "nothing physical". But the theist isn't committed to all causes being physical causes.

    So instead of the conclusion at 5 that the emission is uncaused, we'd properly get a different result: the cause cannot be physical. Therefore, if the emission is caused, it must be a non-physical cause.

    Which just so happens to be exactly the thing many atheists have been demanding out of theists for a while now.

    ReplyDelete
  26. LOL, so basically Rosenberg has destroyed his own view, or gave an argument that destroys his own view?

    ReplyDelete
  27. OL, so basically Rosenberg has destroyed his own view, or gave an argument that destroys his own view?

    If you meant that in reply to me - I'd say so. I note that my response is, for whatever reason, not at all common among theists. Probably because most theists, or at least most debate and philosophy inclined theists, tend to be very reluctant to get into anything that smacks of 'God of the gaps'. And my example would be a textbook example of God of the gaps.

    The problem is, I think 'God of the gaps' is entirely reasonable when the alternative option is 'brute fact' or 'violating causality altogether', etc. I also think the quantum example helps highlight the insincerity of people demanding that sort of evidence.

    ReplyDelete
  28. @OFloinn,
    "Causation does not mean predictability."

    Wouldn't a cause that was unpredictable in principle also mean that it lacked an explanation after the fact?
    If I am permitted to say that "A is the cause of B" then it seems to me that :
    (1) the statement of A entails the statement of B
    (2) the statement of A explains B
    (3) an observation of A may result in the prediction of B.
    The problem as I see it is that certain quantum events do not seem to be on the B end of any of those statements.

    @grodruiges,
    "So what can Rosenberg be appealing to? Ultimately, it can only be to quantum indeterminacy and to some metaphysical interpretation of it. Kick in Craig's response to defuse it."

    I certainly agree that the fact that there exist interpretations of quantum mechanics in which all events have an identifiable cause, and this makes Rosenberg's counterargument sound kind of lame. On the other hand the theory, if true would seem to constrain the form that the causes would have. Are non local, fundamentally unobservable, causes sufficient for our needs? Every defense I have ever heard for the principle of causation sort of suggest that causes would be related to their effects in some way other than simple the brutal fact of the causal relationship. If this is not so then why even bother?

    @Christopher,
    I submit to you that a sufficiently large number of billiard balls has free will. I should also point out that if some physical system is actually indeterminate then they all are, so it is very bad form to both make arguments from the indeterminacy of radioactive decay and the determinacy of the physical brain.

    ReplyDelete
  29. I watched some of that, and found it excruciatingly boring. No progress has been made on settling this question in 2000 years. I am embarrassed that so many intelligent people waste so much time on it.

    Has it occured to anyone that perhaps neither of these sides are right? That we don’t really know what it means for God to exist, and thus it makes no sense to argue for or against it?

    ReplyDelete
  30. @BLS

    "Rosenberg keeps saying that neuroscience proves that there is no intentionality."

    What was his argument? Does he even know what intentionality is?


    He never gave one. I have never heard anyone t claim that neuroscience refutes intentionality so I was shocked and puzzled. Hence the question. :-/

    ReplyDelete
  31. He never gave one. I have never heard anyone t claim that neuroscience refutes intentionality so I was shocked and puzzled. Hence the question. :-/

    It doesn't. Rosenberg has a habit of presenting metaphysical naturalism, even eliminative materialism, with 'science'.

    ReplyDelete
  32. @reighley

    I submit to you that a sufficiently large number of billiard balls has free will.

    How does a billiard ball have will to begin with, let alone free will?

    ReplyDelete
  33. Crude

    Well yeah I sort of understand what you doing, but it doesn't have to be a God of the gaps, it could be simply something not-physical, or a new form of matter XD.

    I mean, there are possibilities.

    -------------------------------------

    Anon

    Yes, the positivists use to say that, the possibility of the impossibility to discuss something does not mean we shouldn't discuss it or that a question is not-answerable.

    Now if you mean that people STILL discuss that after 2000 years and therefore you conclude that things can not be settled... The thing is... this is not as abvious as it goes, I mean is hard to say if everybody that is engaged in the discussion desire to settle it or is even remotely interested in understanding it. Now inside certain schools of thoughts that question is more than settled, either yes or no or something else XD.

    ReplyDelete
  34. How does a billiard ball have will to begin with, let alone free will?
    -------------------------------------

    Well depends on what will is, if will is meant to just be a form of teleology then they would have in Aristoteleanism, teleology. So the balls would have will, maybe just not free will.

    Now if free will is just the possibility to be random, then what reighley means is that a big enough number of billard balls can be taken to be random so they would have free will. (is just that this one is an awkward way XD to define free will)

    ReplyDelete
  35. So Reighley what connect the cause and the effect???

    ReplyDelete
  36. Anonymous: Rosenberg keeps saying that neuroscience proves that there is no intentionality.
    How exactly does neuroscience do that? Anyone have an idea?


    Take neuroscience. Wad it up into a tight little ball, and pack it down the barrel of an improvised cannon, Mythbusters-style. Then fire it at your head and shoot out the part of your brain that processes logical inferences.

    Easy!


    (Oh, and if you criticise my answer, it just proves how much you hate science!!!)

    ReplyDelete
  37. Mr Green

    I am starting to hate science .... XD

    ReplyDelete
  38. reighley,

    Not addressed to me, but just to throw in some comments...

    The problem as I see it is that certain quantum events do not seem to be on the B end of any of those statements.

    I don't think anyone is offering the definitive explanations or list of entailments in quantum theory. In fact, it may be that we're practically unable to discover them. The problem seems to be that the fact that we're faced with a current dead end - maybe even an experimental dead end - doesn't justify us saying 'and therefore, since we can't identify (or experimentally identify) the causes or entailments here, then we should say there are none' or worse 'therefore there are none, and science shows it'.

    ReplyDelete
  39. Wouldn't a cause that was unpredictable in principle also mean that it lacked an explanation after the fact?

    No. We cannot predict which apple will fall on Newton's head or when, but its fall is still caused by "gravity," which we presently conceive as a warpage in the field of Ricci tensors caused by the presence of mass.

    The problem with A and B is that efficient essential causes are typically contemporary with their effects, and in quantum mechanics, so far as we currently know, we cannot know A ahead of time, or at least not completely.

    Chance events happen all the time without bringing causation to ruin. Like the man who is brained by a hammer falling from the roof as he walks to lunch. He was not walking past that point for no reason! It was his lunch time, and he had a favorite diner in mind, and walked at a certain pace. Neither did the hammer fall for no reason: the geometry with which the roofer had laid out his tools, the nudge of his foot as he rose to fetch his own lunch, the coefficient of friction of the roof tiles and the angle of slant on the roof, and finally the force of gravity. (Alas that it was not an apple so dispositioned.) Everything in the sequence of walking lunchbound and falling hammer is caused; yet the whole thing is unpredictable.

    if free will is just the possibility to be random

    But it isn't. A free choice is not a random one; neither is it an unmotivated choice, an irrational choice, an unpredictable choice, or any of the other things so often confused with it. The will is the intellective appetite -- a hunger for the products of the intellect, analogous to the sensory appetites as hungers for the products of sensation. It is impossible to desire or hunger for something which you do not know. Therefore, to the extent that your knowledge is imperfect, your will is not determined to this or that particular means, and so has "play" in the engineering sense, or "degrees of freedom."

    ReplyDelete
  40. I didn't think Craig was as sharp as he is usually, however Rosenberg never posed much of a challenge (and seemed to cede defeat in his opening speech when he actually made an argument against arguments).


    Regarding the kalam, I think what makes it valuable in God/atheism debates is its ability to appeal to modern empirical scientific findings--a source of knowledge which naturalists aren't shy in worshiping. It is fun to see them scramble away from the mild implication Craig draws from the BGV Theorem--merely that the universe began to exist. To the naturalist, Almighty "Science" (what a broad and useless term) is apparently only appealed to when it can be formatted in a way to defend atheistic metaphysics.



    ReplyDelete
  41. Here is my favorite whopper from Rosenberg at 1hr:43min into the debate:

    “The absence of a role for God in the predictive and explanatory content of science is, quite apart from the problem of evil, the primary reason 95% of members of the academy of science in the united states are atheists . . . and why science can provide not only no good basis for theism, but an excellent argument against it”.

    If the methodological ability to conduct one’s chosen profession successfully and efficiently while setting set aside questions about God is actually taken by scientists as a good “reason” for rejecting theism and embracing atheism, then 95% of the members of the academy of science have a philosophical comprehension not worthy of an undergraduate (I don’t think this is true, of course, because Rosenberg’s attribution of this logic to the academy members is insulting and absurd).

    On that logic, plumbers, bakers and candlestick makers have equal grounds for rejecting theism and embracing atheism. As if one’s chosen profession, restricted to a specific dimensions of reality, and explored through limited methodological tools, were ipso facto coextensive with the totality of the real.

    I would have more sympathy for Rosenberg in articulating such nonsense if he were by profession a scientist, but he is a philosopher by trade. His assertion here is sophomoric.

    -Pax

    ReplyDelete
  42. "@Christopher,
    I submit to you that a sufficiently large number of billiard balls has free will. I should also point out that if some physical system is actually indeterminate then they all are, so it is very bad form to both make arguments from the indeterminacy of radioactive decay and the determinacy of the physical brain."

    I don't think that adding an extra billiard ball would contribute at all to a billiard ball system having more free will, anymore than adding more water to a partially filled cup would contribute to the water getting closer to transforming into wine.


    And I share Christopher's concern. If (1) all bodily actions require the activity of neuronal firing in order to be actualized, if (2) the coherency of libertarian freedom (i.e. true freedom) is said to be preserved via the realm of the intellect, and if (3) some of our bodily actions must be truly free in order for us to sensibly be considered free, then shouldn't our intellect (an immaterial entity) influence the firing of neurons (material entities)to a sufficient extent in order to generate an appropriate bodily response, like typing out this post or engaging in a debate? And shouldn't the activity of intellection be the primary, solitary activator of this sequence, with the intellect and its thought process not being the offshoot of some seething neuronal soup, but rather being the wellspring of this sequence of action?


    Right now, I'm not understanding how A-T preserves libertarian freedom. If the intellect isn't primary, and doesn't exert some top-down causation, then how can it have any effect on the unfolding of bodily physiology? It would either be in a separate realm, and/or be the product of bodily physiology.

    ReplyDelete
  43. well be patient, the thomists will show up pretty soon U_U!

    ReplyDelete
  44. Yeah, I know, I've mocked the proliferation of informal fallacies before. But watching that debate causes me to ask if there's an "informal fallacy" termed "Argument from Indignation" or the Rosenberg Fallacy?

    Because if there is not, Rosenberg ought to be delivered credit for best exemplifying it.

    Whatever you think of the "logical problem of evil" for theodicies, Rosenberg's pseudo-philosophical fuming is, on his own terms, just plain stupid ... and even comical.


    I know from reading Rosenberg that intentions are unreal or illusory .. but are emotional outbursts too? His, for example?

    Rosenberg reminds me of a comic relief character in a scene from that comic book like science fiction flick, Starship Troopers who: 'frankly [finds] the idea of a bug that thinks offensive!'

    http://www.youtube.com/watch?v=jVYPxw2oJTg&feature=player_detailpage#t=309s

    ReplyDelete
  45. First, I like Craig, but he has a schtick that he takes on the road every six months. That makes him vulnerable to the kinds of ad hominem attacks that Rosenberg opened up with.

    But, all the same, Rosenberg is pretty pathetic in this debate.

    If this is all they have, we Thomists are going to mop the floor with them. They are sitting ducks.

    (PS: Next time, Alex, get drunk after the debate, not before)






    ReplyDelete
  46. My thanks to Professor Feser for this video link and for the video link in his previous post, Metaphysical middle man, to his 2011 lecture / presentation at Franciscan University of Steubenville (Science and Faith Conference). I find it a bit easier for me to follow the logic in a lecture format than in a live debate format [because of the (perhaps necessary) pace of the live debate format].

    To this general topic (science and faith) I would like to add Addy Pross' book, What is Life? How Chemistry Becomes Biology on Amazon.com here, which I previously cited on other blog posts, including Robert Oerter's blog post, (re his discussion with Tyler of the thirdmillennialtemplar). I find the subject of probabilities and thermodynamics under-reported in these discussions of science and faith (IMO).

    (I hope the links work, because I'm having some Internet connection problems).

    ReplyDelete
  47. Well, we've established that the gnus have no leg to stand on. Okay.

    Now I was in the library and found a book by George. H. Smith titled "The Case Against God". Since I believe most of the questions to be secondary to the proofs of the existence of God, I went and skipped ahead to that section.

    Long story short, I was impressed that, unlike most of the gnus, he took a decent shot at taking down Aquinas's First Cause argument, acknowledging its strengths and making a fair effort to rebut it (although not without some digs in there, but whatever).

    So, here's me laying down the gauntlet. We've more or less established now what the true Thomistic Cosmological argument REALLY is. Now we found a guy who actually tries to take apart the REAL argument and not a strawman version. So, Dr. Feser, what is your response?

    Here's a real test. This was no strawman. It was the actual proof, and he made a nice go at it. What is your response, Dr. Feser?

    ReplyDelete
  48. "Now I was in the library and found a book by George. H. Smith titled 'The Case Against God'."

    For the record, in case anyone isn't familiar with it, the title is Atheism: The Case Against God.

    ReplyDelete
  49. @untenured

    First, I like Craig, but he has a schtick that he takes on the road every six months. That makes him vulnerable to the kinds of ad hominem attacks that Rosenberg opened up with.

    What schtick are you referring to?

    ReplyDelete
  50. @MarcAnthony

    George. H. Smith titled "The Case Against God"

    Is this the same Smith that Bahnsen destroyed in a debate or am I confusing his with someone else? The guy that s considered one of the pioneers of atheistic objections to Theism based on non-cognitivism (which are utter failures as argument by the way). Is this the guy?

    ReplyDelete
  51. I think Untenured is referring to the fact that Craig does have a core group of arguments that he deploys in these debates, by his own admission. The problem is that Rosenberg's response of 'People have refuted these arguments and Craig ignores that!' because A) Craig does respond to criticisms of his arguments in literature, etc, and B) he also responds to them during debate.

    The last part is important, and Craig's actually had a brilliant response to this criticism: he doesn't use any surprises in a debate, by his own admission. He uses the same arguments. If his arguments are easy to knock down, then how come Rosenberg wasn't able to do this?

    ReplyDelete
  52. Don't question Rosenberg, you fools. He is the chair of the philosophy department at Duke University, a rigorous "top ten" university that is respected worldwide. Meanwhile, Craig is merely a dabbling research professor at some fluffy, small, shady Christian college in California (I forget the name, which is telling in itself).

    They are leagues apart in philosophical sophistication and respectability.

    ReplyDelete
  53. I hope this was sarcasm...

    ReplyDelete
  54. I think the most distinguishing aspect of Rosenberg's career is his offering a materialist philosophy other materialists try desperately to pretend does not exist.

    He also, I believe, was part of the Group of 88 in the Lacrosse rape case. Which, honest to God, does not get brought up enough. Did those involved in the group ever issue a public apology?

    ReplyDelete
  55. Reighley,

    On the other hand the theory, if true would seem to constrain the form that the causes would have.

    One thing I find baffling about all invocations of QM in this context is that people seem to think that, until QM, no one had ever realized that some things happen by chance. If I put $100 on red 15 and the ball drops on black, then even if there is a reason why I placed my bet where I did, and a reason why the ball fell on black, there is no coordinating cause explaining why they both happened together. As Aristotle pointed out, an earthquake might happen when you step in the bathtub, but you do not need to look for a reason why both happened at once. You can, of course, figure out the relative probabilities of both happening, and manipulate the resultant numbers however you wish. If earthquakes that occurred while people stepped in bathtubs were something that insurance companies cared about, then they would doubtless keep pretty sophisticated data sets on such contingencies and even predict how much money they would have to budget to cover them - but the event would have no cause all the same.

    Briefly, the principle of causality does not include events that happen purely by chance; and this is true even though chance events, like caused ones, can be predicted and brought under algebraic laws in various ways. If QM wants to say somethings have no causes and are therefore by chance, I don't see why this merits anything more than a shrug. Did they think Aristotle or STA thought nothing happened by chance? (cf. Physics II 4-6)

    ReplyDelete
  56. Emerson

    It just usually depends on exactly is someones theology and what exactly they are talking about that particular theology.

    A-T theology doesn't seem to brush a lot on thermodynamics and probabilties (well unless you thomists think otherwise n_n)

    So dunno it all depends Emerson, although I do agree that such subjects are important to the discussion.

    ReplyDelete
  57. I think the most distinguishing aspect of Rosenberg's career is his offering a materialist philosophy other materialists try desperately to pretend does not exist.

    Exactly!


    ReplyDelete
  58. Is this the same Smith that Bahnsen destroyed in a debate or am I confusing his with someone else? The guy that s considered one of the pioneers of atheistic objections to Theism based on non-cognitivism (which are utter failures as argument by the way). Is this the guy?

    I don't know. What I DO know, though, is that he went after St. Thomas's ACTUAL argument. This is more than the vast majority of the gnus have done.

    I think people familiar with this blog have established that the gnus objections are weak. But there are some intelligent atheists out there.

    I'm not educated enough to know for sure if his objections to St. Thomas were actually philosophically challenging, but I find it refreshing to see somebody go through Aquinas's real argument and not a strawman version.

    ReplyDelete
  59. Eduardo,
    Thanks for the response. I have no positive or negative point to make, just a completeness point I suppose.

    Thinkers sometimes overlook that biologists and physicists had a pretty long philosophic debate themselves (philosophy of science I mean) as to whether biology even qualified as a science in its own right. I can't help but notice that schools still form around a number of key organizing positions: A-T, Catholicism, Protestant faith-based, evolutionary biologists, cell biologists, physicists - I just wanted to bring up probability theory (used by all sides at times) and chemistry (especially thermodynamics because thermodynamics often describes how energy stabilizes in ordered environments - ie counter to the over-riding effect of entropy).

    ReplyDelete
  60. Emerson

    Well I agree overall with you, but I personally see these theories being important for fine-tuning type of arguments. Now apparently classical theism doesn't follow from the typical fine-tuning argument, so that is why I felt that maybe the discussion, although important because some people do have a theology that involves tuning, maybe irrelevant to some other theologies.

    So I feel like these subjects maybe be irrelevant to some people and central to others, got what I was trying to say?

    ReplyDelete
  61. @marcanthony

    I think people familiar with this blog have established that the gnus objections are weak. But there are some intelligent atheists out there.

    I'm not educated enough to know for sure if his objections to St. Thomas were actually philosophically challenging, but I find it refreshing to see somebody go through Aquinas's real argument and not a strawman version.


    The gnus are irrelevant. They are a laughable group that deserve nothing but ridicule.

    Many have criticized Aquinas, the most notable is Anthony Kenny from what I know (and he is an intelligent and respectable agnostic imo). Several Theists including Oderberg have of course responded to his criticism. Smith is not the first one nor will he be the last and I wouldn't be surprised if someone has already responded to his objections although like yourself I would be interested to hear what the Thomists here have to say.

    As far as intelligent atheists, yes there are, but the intelligence only takes them so far. In at least one way (Theologically) they are not. In fact they are quite foolish when it comes to their belief in atheism. I don't say that to insult, I say that because I believe it's true.

    I also checked and Smith is the same guy Bahsen destroyed in the debate. Nowbased on what I heard in the debate and given that it was a complete wash, I don't think much of him.

    ReplyDelete
  62. @James Chastek,
    "Briefly, the principle of causality does not include events that happen purely by chance;"

    Why not? I don't think that the second way can be sustained once we admit that some events do not have efficient causes. Also, the presentation of the exception sounds a little circular to me, almost as though only things that had causes were assured of having causes.

    @Crude,
    "The problem seems to be that the fact that we're faced with a current dead end - maybe even an experimental dead end - doesn't justify us saying 'and therefore, since we can't identify (or experimentally identify) the causes or entailments here, then we should say there are none' or worse 'therefore there are none, and science shows it'."

    I agree, and I agree that arguments from the indeterminacy of quantum mechanics are usually very weak. I do think that the principle of causation is going to need a more detailed defense than Aquinas had to give it. Aquinas knew of no cases in which a cause could not be easily assigned. He was also not concerned with the apparent time asymmetry of the relationship (why should he be?). We know of cases, we should be concerned.

    @OFloinn,
    "No. We cannot predict which apple will fall on Newton's head or when, but its fall is still caused by "gravity," which we presently conceive as a warpage in the field of Ricci tensors caused by the presence of mass."

    Falling apples and hammers may not be predictable in practice, but they are predictable in principle. Let us ask, gravity aside, why the apple fell now rather than later. You have two choices : either you admit that there was something other than gravity in play, so that your model lacked predictive power more by willful ignorance than any mischief by the apple. Or you admit that you just don't know but you are sure there is something, by dint of the principle of causation. In which case the principle of causation will require some stronger defense than "we know of no cases in which causation has been violated".

    ReplyDelete
  63. @reighley
    Let us ask, quantum mechanics aside, why the virtual particle popped up (and immediately disappeared) now rather than later. You have two choices : either you admit that there was something other than quantum mechanics in play, so that your model lacked predictive power more by willful ignorance than any mischief by the particle. Or you admit that you just don't know (due to some sort of Uncertainty Principle) but you are sure there is something.

    Now, given the history of physics, the smart money is on the additional principles. That is why I very carefully said that the timing of the fall of the apple is unpredictable by the theory of gravity. If you examine the equations carefully you will notice that t does not occur as a term in the model. Neither does it occur in the model equations of QM.

    IOW, whether something is caused or not, the timing need not be predictable, at least given the initial primitive models.

    Secondly, the virtual particle is caused by the quantum vacuum. No QV, no VP. That we do not (or perhaps cannot) understand how this is accomplished, we at least have the QM equations that describe what happens.

    In quantum mechanics, particles are only fluctuations in quantum fields, and particle-particle interactions are simply superposition of fields. Causaltiy in its ordinary sense involves the arrow of time – causes come before effects – a notion that has no meaning at quantum level. You must think of it more like the causal relationship of action and reaction.

    Thirdly, as the example of the man being brained by the hammer indicated, there is no causal connection between the hammer and the desire for lunch. The hammer follows its own causal chain. The lunchbound fellow follows his own decision chain. It is chance alone, not causation, that resulted in the two worldlines intersecting.

    There is no causal law that explains passers-by-brained-by-falling-hammers. There is only statistics on how often it might happen; and statistics is not causal.

    ReplyDelete
  64. @OFloinn,
    "Let us ask, quantum mechanics aside, why the virtual particle popped up (and immediately disappeared) now rather than later. You have two choices : either you admit that there was something other than quantum mechanics in play, so that your model lacked predictive power more by willful ignorance than any mischief by the particle. Or you admit that you just don't know (due to some sort of Uncertainty Principle) but you are sure there is something."

    How about for the purposes of exposition I choose that second one, except for the part about being sure that there is something.

    A photon scattered left rather than right and I just don't know why. I think there might not be a reason. I think I have in my hot little hands an event that had no efficient cause.

    Now try defending Aquinas' second way from premises that I can agree to.

    ReplyDelete
  65. Did anyone here notice Rosenberg's comments near the end of the debate where he defended immaterial objects, namely mathematical objects, because of the indispensable role they play in science? Any yet, by his own lights, there can be no such thing as immaterial objects. I imagine that this was just a distraction so he could then throw another immaterial object, namely God, under a bus.

    ReplyDelete
  66. I think there might not be a reason. I think I have in my hot little hands an event that had no efficient cause.

    I think that's a valid move only insofar as explicit and outright God of the gaps is a valid move. Maybe even less so, since at least by leaning on God you're retaining causality rather than deciding to grant exceptions to it based on what amounts to a lack of information.

    ReplyDelete
  67. I thought this might be relevant to some of the chitchat going on here: Newsflash! Some things happen by chance! by James Chastek

    ReplyDelete
  68. I think I am within the arc of this thread, to present the two Q's below (please do not accuse me of blindly opposing A-T logic - I am sincerely interested in any reasoned response and any additional links):

    Prof Feser, in Aquinas: A Beginner's Guide [p.71-72, relying on Aquinas], says, "If a first member who is the source of the causal power of the others did not exist, the series as a whole simply would not exist .... What Aquinas is saying, then, is that it is in the very nature of causal series ordered per se to have a first member, precisely because everything else in the series only counts as a member in the first place relative to the actions of a first cause. To suggest that such a series might regress infinitely, without a first member, is therefore simply unintelligible" [italics added]. At this spot it is important for me to say, I am aware that Feser has already distinguished between a causal series ordered per accidens, which is "in Aquinas' view at least theoretically possible" [p.71] and a causal series ordered per se.

    I am very interested in this distinction. My Q is, (1) why is it "unintelligible" that a series per se "might regress infinitely"? [p.72] and (2) if it is unintelligible, then why is God the answer, rather than an eternally existing natural force?

    Aren't these the two central points of debate between Craig and Rosenberg?

    BTW, I am also aware of the distinction between act and potency, but it would certainly be acceptable to answer my Q's in terms of why it would be "unintelligible" for an eternally existing natural force to be pure act, in the same manner that God is considered to be pure act. Peace.

    ReplyDelete
  69. Kiel,
    Thanks for the new link. Good reading.

    ReplyDelete
  70. Well I think the answer to your second question is along the lines that, people have carefully tried to show that the first cause has divine attributes, attributes that usually we consider God to have, so your questions is a Book-long XD answer, but the major is that: They argue for the other attributes that distinguish it from a "natural" force.

    The word Natural in A-T is rather different then we usually use in modern days, that is also worth mentioning.

    I think the first question has a shorter answer with the argument to back it up being somewhat big, but I do remember people talking about it somewhere in here.

    -----------------------------------

    Now this is also worth thinking about and who knows maybe doing some research of your own, but Leibniz main idea was to prove that there are no such things as brute facts, and hence no "divine" natural (in the modern sense here) thing.

    So maybe you can't use any natural force to replace a let's say, Deistic God.

    ReplyDelete
  71. My Q is, (1) why is it "unintelligible" that a series per se "might regress infinitely"? [p.72] and (2) if it is unintelligible, then why is God the answer, rather than an eternally existing natural force?

    TheOFloinn has provided the image of our times that helps explain this: an infinitely long forwarded email chain. It simply would be "unintelligible" to say that such a chain regressed infinitely and did not require as a first member, some thing other than a forwarded message, that actually provides the potentially infinitely long series with its content. And we would be right in thinking that this first member is radically different from a forwarded message. Alternatively, the image of a series of new/ reply messages would be a series per accidens because the content of each message is independent of the other, whereas a forwarded message is entirely dependent on the first in the series.

    This really is a wonderful illustration, OFloinn.

    ReplyDelete
  72. Yes, perhaps Q(1) is sufficient here, as it relates to the cited Feser text. But I can't help pointing out that one of the things that drew me to this post in the first place was that the second commenter quoted Craig (at about 15:30 in the audio) as ending his first argument with the words, "... which is what everybody means by God." Now I realize that arguments for the attributes of God fill volumes, but still ... Aquinas did not choose to conclude the Five Ways with something along the lines of "the Central Principle of Nature." Thanks for replying.

    ReplyDelete
  73. dover_beach,

    Yes, but is the acceptance of this "unintelligibility" the result of the compulsion of our system of logic (in other words, there is no alternative except to be irrational), or is the acceptance of this "unintelligibility" the result of an inductive generalization or "best explanation". Craig used the latter, "best explanation", form of argument. It seems to me that "best explanation" leaves room for legitimate disagreement.

    ReplyDelete
  74. Well, could he? Given his metaphysics?

    I think Aquinas could have said that, but the thing is: He is trying to prove God's existence, so it makes more sense to allude to a definition of God then God's relation towards reality or nature, which would sound the books more like a project of how reality is, more towards Natural Phylosophy instead of Theology.

    ----------------------------------

    Now about the emails, exactly what type of relations can we consider something to be intelligible.

    I mean I get the outer bounds of the idea, but not the core of the idea.

    ReplyDelete
  75. I' d have to listen to that section again but it seems to me if you're concerned with intelligibility, you're not concerned with an argument to the best explanation. You may be confusing his fine-tuning argument to this first argument.

    ReplyDelete
  76. I'm interested in how you all see Rosenberg? I see him full of anger.

    ReplyDelete
  77. I don't think ROsenberg cares about my beliefs about his overall behavior... after all for him, there are no beliefs XD.

    ReplyDelete
  78. @Crude,
    "at least by leaning on God you're retaining causality rather than deciding to grant exceptions to it based on what amounts to a lack of information."

    Would you say that you agree with @TheOFloinn's statement that "There is no causal law that explains passers-by-brained-by-falling-hammers."

    or with @James Chastek that "the principle of causality does not include events that happen purely by chance;"

    For I fear that they are admitting of exceptions to causality, and I do not think there can be any exceptions if the principle is to be of any use to anybody. This is the point I am trying to make above : just grant me one little photon scattering this way or that and the whole logical ship begins to founder. I cannot draw inferences from a rule with even one exception.

    ReplyDelete
  79. Wow, re-watching the debate. Rosenberg is just parroting party-line naturalism; the stuff that everybody just mindlessly repeats in every graduate seminar with the expectation of going unchallenged. I don't even think he listened to Craig's second set of arguments.

    These people have no idea what they are up against; absolutely none. They think Theism is so silly they can just phone it in and not even try.

    ReplyDelete
  80. Reighley,

    If I may; I don't want to speak for either, but it seems to me that what both James C. and TheOFloinn are stating is the traditional Thomistic notion that "chance", considered ontologically, is nothing other than the meeting of two or more causal series. No doubt, that is a different notion of "chance" than is popularly held. But that is because those who use the word in a popular sense often fail to think through its ontological status. IOW, what IS "chance" as an extra-mental reality? But if "chance", by definition, is what Thomsits say it is, then there is nothing within the notion of "chance" that dos not take its fundament from causality.

    Hence, "chance" in no way undermines the ontological ubiquity of causality (i.e. it does not slip in another ontic category among existents). To the contrary, an ontological focus upon the notion of chance leads one to a stronger affirmation of causality; for "chance", as a concept, has no ontic existence outside of an interlocking framework of causation. Nothing about QM (nor inertia) undermines the reality of that causal framework.

    Therefore, the principle of causality (which is a mode of the principle of sufficient reason) remains a live princple within the philosophy of nature and hence, metaphysics; and in particular, natural theology. If neither QM, nor inertial theories undermine the principle of causality, the traditional theistic arguments continue to go through.

    -Pax

    ReplyDelete
  81. @reighley

    A photon scattered left rather than right and I just don't know why. I think there might not be a reason. I think I have in my hot little hands an event that had no efficient cause.

    But ignorance about an efficient cause does not logically entail that there is no cause.

    Here's an idea...

    The form of the QV out of which the VP pops out has a structure that facilitates probabilistic effects in contrast to say deterministic effects. In more detail, imagine that the QV structure operates by the principle of generating numbers/values. One or more of the numbers/values is the number that am its the VP. So if say the value is 99, once the generation of that number hit 99 the particle pops if it doesn't then it will not. By virtue of this process being by chance (not true randomness) the principle of causality is obviously at play but it's not deterministic. It's probabilistic.

    ReplyDelete
  82. Stupid auto-correct...

    One or more of the numbers/values is the number that EMMITS the VP.

    ReplyDelete
  83. IIRC, at least we do know why the photon scatters at all, and that it is due to its wave-particle nature combined with a physical thin slit. So while it may be an "incomplete" efficient cause, its not like there is NO efficient cause in the event of scattering. If I emit a photon into an infinite vacuum, I don't think it is going to scatter in any direction unless it passes through a thin slit.

    ReplyDelete
  84. Crude said...

    I think the most distinguishing aspect of Rosenberg's career is his offering a materialist philosophy other materialists try desperately to pretend does not exist.

    He also, I believe, was part of the Group of 88 in the Lacrosse rape case. Which, honest to God, does not get brought up enough. ..
    February 4, 2013 at 3:40 PM "



    Observing his manner on the podium, I have no trouble believing you regarding his being part of the gang of 88 lynch mob.

    Since this blog's author comments on ethics as well as metaphysics, it might be worth taking a look at how Rosenberg's metaphysics seems to have shaped his ethics.

    When Googling the matter I came across this:

    "Ironically, the most extreme manifestation of groupthink has come from one of the few Group of 88 members whose research eschews race/class/gender issues. Philosophy's Alex Rosenberg is the only signatory to have had Reade Seligmann in class. I e-mailed Rosenberg, said that I had blogged about the case, and asked ..."

    http://durhamwonderland.blogspot.com/2006/07/open-letter-to-brodhead.html
    and
    http://johnsville.blogspot.com/2006/11/duke-case-listening-statement.html

    I do not think that Rosenberg's bad character can refute the logic of any particular argument he might construct, but I do think that his sneering manner of arguing accurately reflects his character, and reflects something deeper about any commitment to truth he might claim.

    ReplyDelete

  85. These people have no idea what they are up against; absolutely none. They think Theism is so silly they can just phone it in and not even try.


    Well...yes.

    Theism is pretty silly, especially the philosophical versions of it. can accept belief when it is part of actual spiritual practice, but as an intellectual belief it is pretty pathetic excuse for thinking.

    So although I am more on Rosenberg's side than otherwise, I think both parties to this are pretty lame, as am I for paying attention to it. Why don't we all just agree that God == the unknowable and go about our business?

    ReplyDelete
  86. Grodrigues said,

    " "(1) Put two lumps of radioactive material, A and B, side by side.

    (2) They are identical in all physical features.

    (3) At time t, A emits an alpha particle and B does not.

    (4) We can hear now Rosenberg ask rhetorically, what can account for the difference? Why nothing of course, because of 2 and B did not emit any alpha particle.

    (5) Ergo, the event "A emitted an alpha particle" is uncaused.

    As the TheOFloinn, indeterminacy is *not* lack of causation, but let us forego that for the sake of argument. It still does not follow what Rosenberg thinks it does. For what does (2) mean? Remember that Rosenberg insists that it is a matter of *experiment*. It is impossible, even in principle, to have two identical lumps of radioactive material so (2) is impossible to replicate in experimental practice. Maybe Rosenberg will concede and then say, forget the lumps and take just two radioactive atoms. But (2) is still impossible to obtain, even in principle, for among other things, they will be different in space-time location or in some other accident for otherwise we would not know that they are different. Heisenberg's principle says that it is impossible for us to have an experimental setup with two identical atoms even with respect to energy or other similar observables. So what can Rosenberg be appealing to?"



    Well stated, and just so. OFloinn, Crude and several others have offered perceptive questions and comments into the problematic of just how you would be expected to unpack what Rosenberg's statement to the effect that these are the same atoms (at one point he says molecules) or no different, would mean.

    The notions of time, age, and location, and even "two", would all seem to make the idea of "the same", rather than "the indistinguishable", problematic.


    Then too, there are, as others have mentioned, action or effect at a non-obvious distance, and the problematical conceptualization of these particles. At the least therefore, I don't see how Rosenberg can make the claim he does - or extend it as he does - in good faith.

    I have referred to my undergrad texts in physics, such as they were, as well as spent a couple of hours poring over the infamous Wiki entries on indeterminacy, uncertainty, observer effect, and the like, and can see no reason for Rosenberg's blithe dogmatism on this point.

    Well no reason than an agenda ...

    ReplyDelete
  87. Right if you found that God exists.... Obviously is something worthless. And you obviously can't derive any practice too O_O!

    Sorry Anon, but accepting God as the unknowable without any argument just a gut feeling is pretty damn silly too! Welcome to the club!!!

    ReplyDelete
  88. "Why don't we all just agree that God == the unknowable and go about our business?"

    Well this just begs the question against those who do believe God can be known through reason.

    ReplyDelete
  89. Reighley,

    Why not? I don't think that the second way can be sustained once we admit that some events do not have efficient causes. Also, the presentation of the exception sounds a little circular to me, almost as though only things that had causes were assured of having causes.

    The second way starts by taking a series of efficient causes as a given. It does not say that there is an efficient case for everything, including things that happen by chance. Both St. Thomas and Aristotle speak at great length about the reality of chance: see here. or the four chapters starting here.

    I wouldn't deny that chance is in some way a cause, since "cause" can be taken broadly enough as to include any explanans and we do cite chance to explain why something happened. But even if chance is understood as a cause in a broad and accidental sense, it still has to be understood through the negation of causal connection, and so our knowledge of it will be one of the substitutes we use when we can't know the causes of things, e.g. the use of algebraic natural laws or relations of probabilities.

    There is no circularity here: we can draw a sharp division between things that are generated by chance and those not, namely, that what is generated by chance does not come from a pre-existent form or pattern. St. Thomas gives the principle in the second paragraph of his response here. The distinction is not arbitrary or dashed-off - the same persons that invented and refined conmological arguments spoke about chance at great length (again, see the second link).

    ReplyDelete
  90. @dover_beach:
    I think I picked it up from James Chastek once upon a time.

    I recollect also the Pythagorean Theorem, which you and I and everyone we have ever known has received from a teacher. But each of these teachers was in turn taught by another: a "taught teacher." But such a chain must have a primary cause, even if it is infinite. (Remember, "primary" does not necessarily mean "first in time," although it often does.) This would be a teacher of the Pythagorean Theorem who was not taught by another, an Untaught Teacher, who is the efficient cause of the Theorem regardless how many instrumental (secondary) causes by which it is transmitted to us. Let's call that First Cause "Pythagoras." (An efficient cause is one that brings existence [esse, hence, esse-ntial] to the effect.)

    Another analogy is Mozart's Clarinet Concerto in A, the immediate cause of which is the clarinet. But the clarinet has no power to produce the music unless it is given that power by another; let's say by Sharon Kam. (http://www.youtube.com/watch?v=xr3aB4v8hXI) In an essentially ordered series, each secondary (instrumental) mover receives its power to actualize from a predecessor in the series. If there were no primary actualizer, none of the elements of the series would have any actualizing power at all, and nothing would happen.

    It's like a train of boxcars observed while moving down a track. None of the boxcars has any power in itself to move another, and so the train must have been set in motion by a primary actualizer. (In US RR lingo, it is actually called a "motive".)

    ReplyDelete
  91. reighley:
    For I fear that they are admitting of exceptions to causality, and I do not think there can be any exceptions if the principle is to be of any use to anybody.


    I dunno. Division is of tolerable use, even though it exempts X/0. Gravitational theory is of tolerable use, even though it makes an exception for electromagnetism.

    But pay attention to the hammer (lest you be brained). There is a cause for the poor fellow being brained. He was struck on the head by a falling hammer. And there was a cause for him being struck in that he was headed by intention to his favorite diner and had been walking as such and so a pace as to bring him under the hammer's trajectory at just the wrong moment. Everything in both world-lines, hammer and lunchbound, is caused.

    It is only that there is no cause as to why that particular person was struck by that particular hammer at that particular time. IOW, the nature of causation does not demand predictability. "Chance" is not a "cause." Otherwise, one gets sucked into the superstition of concatenation.

    That A happens and B happens does not imply a causal connection between A and B. A might have been caused by X and B by Y, and it is mere coincidence. Even if A and B alike were caused by X, we have mere correlation, and there is no causal arrow linking A and B. Statisticians are generally familiar with this issue and must constantly remind social "scientists" and even sometimes real scientists that "correlation is not causation." What do they suppose we meant by that?

    If I may impose on Dr. Feser's patience, I will post a statistical anecdote that illustrates this.

    ReplyDelete
  92. Here is the threatened anecdote.

    Once upon a time a QC tech who had taken enough statistics classes to be dangerous noticed a strong correlation between the pressure in a fermentation vat and the yield of the chemical reaction. Every time the yield was low, the pressure was high.

    He went to the superintendent of fermentation and said, "I know how to increase your yields."

    The superintendent said, "Tell me this thing, for I am always anxious to improve my yields."

    "Just lower the pressure in the vessel," the tech responded.

    "Please turn around and bend over, so that I may properly thank you," the superintendent said; for he knew something the technician did not.

    Namely, when there was a certain impurity in one of the raws, it caused a side-reaction, reducing the yield of the primary reaction. That same impurity caused foaming in the vessel and to keep the foaming down, the SOP instructed the operator to increase the pressure.

    So, yes, whenever the yield was low the pressure was high. But there was no causal connection between the yield and the pressure, only a correlation.

    But notice that this does not mean there is no causation. (Down, Hume! Down, boy!) The low yield had a cause (the impurity in the raw material gumming up the chemical reaction). The high pressure had a cause (the operator adjusted the pressure in response to the SOP for controlling foam). They were correlated because they shared a common cause (the impurity caused the foaming, too) and correlation can be thought of as a measure of the percentage of causal factors which two effects have in common.

    ReplyDelete
  93. Well this just begs the question against those who do believe God can be known through reason.

    People in a philosophical discussion should be more careful about throwing around terms like “beg the question” without knowing what they mean.

    But anyway -- I am not impressed with reasoned arguments for God (and no, that in itself is not an argument). They all seem to rely on very dubious presuppositions. But all that aside, even if you accept them, what do you get? You get the Unmoved Mover, the Ground of Being, the Absolute, or something equally airless and abstract. None of those things seems to have much to do with the personal God of the Bible or of direct spiritual experience.

    ReplyDelete
  94. >People in a philosophical discussion should be more careful about throwing around terms like “beg the question” without knowing what they mean.

    So what does "beg the question" here mean? Come on stop begging the question.:-)

    >But anyway -- I am not impressed with reasoned arguments for God (and no, that in itself is not an argument).

    Then you have no standard to believe or disbelieve anything & thus nothing to contribute here.


    >They all seem to rely on very dubious presuppositions.

    We won't hold our breath waiting for you to articulate which specific presuppositions and why are they dubious.

    >But all that aside, even if you accept them, what do you get? You get the Unmoved Mover, the Ground of Being, the Absolute, or something equally airless and abstract. None of those things seems to have much to do with the personal God of the Bible or of direct spiritual experience.

    Only if you A Priori assume the Unknowable Transcendent God of the Bible is an overly anthropomorphic Theistic Personalist post enlightenment type of "deity".

    I have no good reason to believe the God of the Bible and the Unmoved Mover, the Ground of Being, the Absolute etc are not the same God.

    ReplyDelete
  95. "But all that aside, even if you accept them, what do you get? You get the Unmoved Mover, the Ground of Being, the Absolute, or something equally airless and abstract. None of those things seems to have much to do with the personal God of the Bible or of direct spiritual experience."

    You must be new around here.

    ReplyDelete
  96. People in a philosophical discussion should be more careful about throwing around terms like “beg the question” without knowing what they mean.

    From the wikipedia: The fallacy of petitio principii, or "begging the question", is committed "when a proposition which requires proof is assumed without proof"; in order to charitably entertain the argument, it must be taken as given "in some form of the very proposition to be proved, as a premise from which to deduce it".

    You can quibble whether or not saying 'God is unknowable so let's go about our business' is question begging. You certainly didn't require proof for what you claimed. Even if it's not question-begging, it is... well, pretty dumb.

    They all seem to rely on very dubious presuppositions.

    Yeah, that crazy 'ex nihilo nihil fit' and all that. Feel free to argue against them, but that seems to be the exact thing you don't want to do.

    But all that aside, even if you accept them, what do you get? You get the Unmoved Mover, the Ground of Being, the Absolute, or something equally airless and abstract. None of those things seems to have much to do with the personal God of the Bible or of direct spiritual experience.

    Oh gosh, all they do is prove the existence of God. But not the specific God of a particular religion, which would require additional argument, so it must be uninteresting.

    Science must bore the hell out of you.

    ReplyDelete
  97. "Oh gosh, all they do is prove the existence of God. But not the specific God of a particular religion, which would require additional argument, so it must be uninteresting.

    Science must bore the hell out of you."

    I actually LOL'ed at the first part, but I think the science comment is besides the point.

    But yeah, it seems like atheist/agnostic Anon knows a bit about philosophy, so I don't think it is too much to ask for an argument on his/her part.

    ReplyDelete
  98. BLS,

    I actually LOL'ed at the first part, but I think the science comment is besides the point.

    I think if someone can honestly say that the mere demonstration of the existence of God, the Prime Mover, and so on is boring and uninteresting, then it stands to reason quite a lot of science is going to be judged the same way as well.

    ReplyDelete
  99. Actually Anon did throw 1 proposition.

    "Theism without religious practice has no value"

    It seems that what matters to Anon, is some kind of relationship, personal relationship with God.

    But I am detecting all sorts of possible screw ups along the way xD.

    ReplyDelete
  100. @anon

    Theism is pretty silly, especially the philosophical versions of it. can accept belief when it is part of actual spiritual practice, but as an intellectual belief it is pretty pathetic excuse for thinking. 

So although I am more on Rosenberg's side than otherwise,

    You poor thing. I use to think in similar ways back when I was still enslaved by my materialistic commitments. The sole reason you are saying what you are is due to ignorance and nothing more.

    While I maintain that agnosticism is the only other intellectually sound position (other than broad Theism – which includes Deism, Theisms of all sorts, Panentheism etc) the fact of the matter is not only is Theism intellectually defensible it is in fact the foundation of intellectuality. So when some anti-intellectualist likes you spouts nonsense as you have, forgive us for not taking you seriously.

    People in a philosophical discussion should be more careful about throwing around terms like “beg the question” without knowing what they mean.

    Your reasoning (or lack thereof) was circular and you got exposed.

    I would also mention that people should be carful before throwing around nonsense such as “theism is silly”.

    I am not impressed with reasoned arguments for God

    Truth is not concerned with impressionability. Welcome to philosophy 101.

    They all seem to rely on very dubious presuppositions.

    Dubious? Really? Like change is dubious? Or causality is dubious? Or that the world is intelligible? Or that ethics are real and not some delusional practice by a “great ape”?

    If you want to be taken serious, you need to act like a serious person. Right now you’re coming off as a clown.

    But all that aside, even if you accept them, what do you get? You get the Unmoved Mover, the Ground of Being, the Absolute, or something equally airless and abstract.
    Well since you brought up The Absolute I will simply mention Hegel’s argumentation, who concluded that the Absolute is more Real than you and me. So if you’re using the word abstract as something that is less real or somehow compromised, you’re wrong. If you mean you can’t touch, see, feel, hear, taste it… The response is, so what?

    Or perhaps you want to discredit mathematics too perhaps? LOL

    As far as Theology being uninteresting, that’s a preference. I have no problem allowing that much of you, but I personally find it not just interesting but the MOST interesting discipline bar none.

    I also maintain, much like Stephen Barr and the Classics and Medievals that Theology/Metaphysics is the highest discipline followed by philosophy with science coming in third.

    Sure I like my microwave, my iPhone and my MacBook Pro and internet and all that jazz, but what are those things in comparison to a glimpse of the divine? To me this is not even a contestable issue.

    A mere glimpse even if partial into the Divine is the human achievement par excellence!

    ReplyDelete
  101. dover_beach
    > I' d have to listen to that section again but it seems to me if you're concerned with intelligibility, you're not concerned with an argument to the best explanation.

    I'm actually concerned deeply with both types of argument, but you are right, in that I unintentionally conflated Feser's argument in Aquinas [p.71-72] as to "unintelligible" with Craig's argument in the video as to "best explanation". My bad. Nice catch.

    Craig says this [at ~13:30]: "I believe that God's existence best explains a wide range of the data of human experience. Let me mention just eight. First, God is the best explanation of why anything at all exists."

    Craig proceeds with a thought experiment of a hiker finding a ball in the forest. He says that people want and need an explanation of the ball's existence, and that this remains so if the ball is increased to the size of the Universe, which he defines as "all of space time reality".

    He then says [at ~14:30], "The explanation of the Universe can lie only in a transcendent reality, beyond the Universe, beyond space and time, which is metaphysically necessary in its existence. Now there is only one way I can think of to get a contingent Universe from a necessarily existing cause and that is, if the cause is a personal agent who can freely choose to create a contingent reality. It therefore follows that the best explanation of the existence of the contingent Universe is a transcendent personal being ... which is what everybody calls God. "

    I have already said (above) that live TV debates are perhaps a difficult setting to expound clear arguments, so I realize that Craig appears to be aiming at Aquinas' First or Second Way -- and that Feser's argument in his book is more thorough. But isn't Craig pointing at an inductive best explanation of "a wide range of the data of human experience" type of argument -- which leaves Rosenberg room to reasonably disagree?

    And isn't Feser [explaining Aquinas] grounding his /Aquinas' argument on an axiomatic premise that the human mind [indeed any mind] cannot reasonably conceive [or maintain or hold] that a causal series ordered per se can infinitely regress? ... to which someone such as Rosenberg might reasonably respond that he indeed can so conceive?

    Doesn't this correctly frame the debate?

    ReplyDelete
  102. If someone could clarify a few very simple details for me, I'd be really appreciative.

    Generally, I want to know this: when something has two or all of Aristotle's four causes?

    If I was to answer my own question, it seems everything which we experience through the senses has a formal and material cause. But, only those things which participate an a cause/effect relationship have a efficient and final cause (since efficient cause is unintelligible without final cause). This is another way of saying that not everything always has a efficient and final cause.

    Is this an accurate answer to my own question? If not, what do you suggest is a better answer?

    I can think of one objection to my answer: if everything which we experience through the senses participates in an essentially ordered causal series to have existence at all, then it follows that everything we experience through the senses has all four causes.

    However, perhaps the answer to the question depends on perspective. Some description of change might be more fundamental to another description of change.

    Or am I mixing accidentally ordered causation with essentially ordered causation?

    Thoughts?

    ReplyDelete
  103. If I was to answer my own question, it seems everything which we experience through the senses has a formal and material cause.

    Should read:
    If I was to answer my own question, it seems everything which we experience through the senses always has a formal and material cause.

    ReplyDelete
  104. What reason is there for thinking that the prime mover is the same as the childish chap in the Bible who is obsessed with how people have sex, constantly commands genocide (Canaanites & co) and gets so angry if people commit thought crime (become unbelievers) that he has to torture them for all eternity in hell. There seems to be a world of difference. One seems to be the sort of entity that could command the respect of thinking people today. The other sounds like a monstrous creature birthed by primitive barbarians.

    ReplyDelete
  105. I would like to hear answers to Kiel's formulation of the questions. I too am trying to understand this same distinction. Thanks.

    ReplyDelete
  106. Serious question guys: What on earth makes you think the bizarre absurdity depicted in the Bible should be identified with the ground of being?

    ReplyDelete
  107. I don't intend to buy Feser's book so don't waste your time. It's ok if you didn't understand his argument and don't feel competent to discuss it. But if you actually understood what he was saying I encourage you to present his views here.

    ReplyDelete
  108. me: They all seem to rely on very dubious presuppositions.

    other anon: Dubious? Really? Like change is dubious? Or causality is dubious? Or that the world is intelligible? Or that ethics are real and not some delusional practice by a “great ape”?

    I’m not sure exactly what argument you are making -- I suppose that the existence of (pick one or more) change, causality, intelligibility, or ethics proves the existence of god. I am aware of these arguments, and they are all extremely unconvincing to those who are not already convinced. And likewise, naturalists and atheists have theories of these phenomena that are completely unconvincing to theists. This is why, despite all these ironclad arguments, nothing about this issue has been settled in 2000 years.

    So you entirely missed my larger point, which is that I find both sides of this debate equally a waste of time. I’ve made some naturalist arguments here in the past, but I’m not doing so now, because what’s the point? A naturalist theory of ethics (eg) is of intense interest to naturalists, but will just cause harrumphing from those who are predisposed not to accept it.

    ReplyDelete
  109. @anon

    What reason is there for thinking that the prime mover is the same as the childish chap in the Bible who is obsessed with how people have sex, constantly commands genocide (Canaanites & co) and gets so angry if people commit thought crime (become unbelievers) that he has to torture them for all eternity in hell. There seems to be a world of difference. One seems to be the sort of entity that could command the respect of thinking people today. The other sounds like a monstrous creature birthed by primitive barbarians.

    Yes, there are some strange stories in The Bible, even ones that are disturbing but not all depictions of God in The Bible are of this sort.

    You must always remember that The Bible was written by many different people, each of which has a different way of seeing and expressing their thought.

    Here are a few passages:

    (Deuteronomy 3:24 NIV) "O Sovereign LORD, you have begun to show to your servant your greatness and your strong hand. For what god is there in heaven or on earth who can do the deeds and mighty works you do?

    This passage signifies the awe and wonder we all have when we look at the world. This is precisely the religious instinct Einstein was so much connected to.

    (Matthew 5:48)Be perfect, therefore, as your heavenly Father is perfect.

    This is nothing short of the God of the Philosophers.

    (Psalms 46:1-2 NIV) God is our refuge and strength, an ever-present help in trouble. {2} Therefore we will not fear, though the earth give way and the mountains fall into the heart of the sea,

    The denial of the atheistic despair.

    (Luke 17:21 NIV) nor will people say, 'Here it is,' or 'There it is,' because the kingdom of God is within you."

    God as a spiritual reality that attests to the experience of common man.

    God is spirit, and his worshipers must worship in spirit and in truth."

    The rejection of materialism (not the metaphysic but the preoccupation with material things) and the prompting for man to actualize his full potential (spirit) and seek truth.

    For the kingdom of God is not a matter of eating and drinking, but of righteousness, peace and joy in the Holy Spirit,

    To know God is to know joy and peace.

    And so we know and rely on the love God has for us. God is love. Whoever lives in love lives in God, and God in him.

    I will not comment in this last one as to comment would be to detract from it. ;-)


    See? That's a very different aspect of God as you can see. An image that is not only appropriate for modern and post-modern man, but one without which man falls into nihilism.

    ReplyDelete
  110. Well anon, people don't care lol. What is your opinion on the matter, they want you to give an argument, but if think it is all worthless, I welcome you to stand aside.

    ReplyDelete
  111. Anonymous said...
    I don't intend to buy Feser's book so don't waste your time. It's ok if you didn't understand his argument and don't feel competent to discuss it. But if you actually understood what he was saying I encourage you to present his views here.

    February 5, 2013 at 5:27 PM


    Ok, well try these YouTube videos mentioned in this post. I needed to watch them a handful of times for his points to sink in.

    Obviously I'm still in the process of perfecting my understanding so I do not feel comfortable giving you an answer. I am also not going to answer your question by electronically regurgitating Feser's book.

    If you value truth, have integrity and take the Christian claim seriously, I think it is reasonable you seriously consider buying the book (or at least try to find somewhere to borrow it). It will be worth it because it will at least help inform your belief, even if you disagree.

    ReplyDelete
  112. One anon: "They all seem to rely on very dubious presuppositions."

    Other anon: "Dubious? Really? Like change is dubious? Or causality is dubious? Or that the world is intelligible? Or that ethics are real and not some delusional practice by a 'great ape'?"

    First anon: "I'm not sure exactly what argument you are making -- I suppose that the existence of (pick one or more) change, causality, intelligibility, or ethics proves the existence of god."

    You said the arguments for God's existence all seemed to you to reply on very dubious presuppositions. The other anon is pointing out that some of those arguments rest on resuppositions like the occurrence of change, the existence of causality, the intelligibility of the universe, and the meaningfulness of ethics. The implied point is that, if you meant what you wrote, then you must regard those presuppositions as "very dubious," and the express question is whether you in fact do.

    ReplyDelete
  113. @anon

    I’m not sure exactly what argument you are making -- I suppose that the existence of (pick one or more) change, causality, intelligibility, or ethics proves the existence of god.

    You claimed the presuppositions were dubious and I gave you ones that were not to demonstrate you are wrong.



    I am aware of these arguments, and they are all extremely unconvincing to those who are not already convinced.

    Wrong again, first I will speak for myself as someone who was a naïve materialist. Those arguments completely transformed my view of reality. It went from being 2 dimensional and a boring black&white to three dimensions and full color.

    There are many people who have been convinced of these, off the top of my head I shall name Mortimer Adler (who became fascinated with Aristotle’s arguments), CS Lewis who went from atheist to Theist to Christian. Anthony Flew (one of the most noteworthy, if not THE most noteworthy atheist apologists of the 20th century) went from atheism to Deism, Fred Hoyle from atheist to Deist. There are a million other case. You simply happen not to be one, which is fine. But don’t generalize your case.


    And likewise, naturalists and atheists have theories of these phenomena that are completely unconvincing to theists.

    Not really. Not the ones that I mentioned at least. Also keep in mind, naturalism apart from the bandwagon has nothing going for it. It’s been show time and time again to be absurd.

    Let’s not pretend that Theism is on the same level as naturalism, intellectually speaking. Theism is head and shoulders above atheism.

    This is why, despite all these ironclad arguments, nothing about this issue has been settled in 2000 years.



    I think it has been long settled. In fact at this point in my intellectual development I don’t even think atheism/materialism/naturalism/nihilism is even a live option. I am so beyond these beliefs that they seem like nothing but a superstition.

    So you entirely missed my larger point, which is that I find both sides of this debate equally a waste of time.

    Then why are you here wasting more time? I don’t think I’ve missed it. In fact I acknowledge and welcome agnosticism and I think it does one good to be at least a little agnostic just to be reminded that as humans we don’t know it all.


    I’ve made some naturalist arguments here in the past, but I’m not doing so now, because what’s the point?

    You are more than welcome to make another one. But if it’s going to be some petitio principii that appeals to Ockhams razor or some sleigh of hand that tries to conflate the methodology of science with ontology, or some “argument” for non cognitivism I’m afraid you will fail. I can of course raise insurmountable barriers to even there be a possibility for a naturalistic argument since the fundamentals of Reason and truth-seeking are at the center of Theism. But I will not push for that now.

    If you truly think that you can prove naturalism (the proposition that nature is all there is and here is the proof), I’d be glad to hear it. I’ve asked this of many atheists and not a single was able to provide a proof.


    A naturalist theory of ethics (eg) is of intense interest to naturalists, but will just cause harrumphing from those who are predisposed not to accept it.

    I’m with nietzsche on this one. Given atheism, nihilism is the logical conclusion and all ethics are reduced to power struggles. If you can prove him wrong I suppose you might have a possibility of launching an argument.

    I personally don’t think there can ever be a naturalistic ethic that is not arbitrary, incoherent or self-undermining. At least from what I read I have no reason to think otherwise.

    ReplyDelete
  114. they are all extremely unconvincing to those who are not already convinced.

    Indeed, in the Late Modern Ages, following the Triumph of the Will, anything that appeals to the intellect is likely to be met with the dull, stolid gaze of a peasant.
    + + +

    @Kiel
    I can only offer my own poor understanding. Namely this:
    Matter and form are principles that explain a thing's being.
    Agent and End are principles that explain a thing's becoming. Kinesis or "motion" is not only local motion, but includes all actualizations of potentials. Thus, a thing might be growing, aging, ripening, etc.

    ReplyDelete
  115. @TheOfLoin

    Indeed, in the Late Modern Ages, following the Triumph of the Will, anything that appeals to the intellect is likely to be met with the dull, stolid gaze of a peasant.

    Do you mind elaborating a little on that? I'm particularly interested in the historical development of how the Intellect was abandoned and the Will was throned in the mind of modern man.

    I'd be happy to hear more about this. As much as your time and interest cares to share. I'm intrigued!

    (This is not the anon you were responding to by the way)

    ReplyDelete
  116. c emerson, I think that Craig is just being a little loose in his choice of words (forgivable in a verbal engagement) but, as he says, he is giving a metaphysical demonstration, not empirical reasons for preferring this explanation as opposed to another plausible explanation. And let us not confuse the empirical premise of each of these arguments with the argument itself. The First Way, for instance, includes an empirical premise but the argument itself is metaphysical, not empirical. Which is one reason why atheists/ materialists who, confronted by its shear power and reasonableness, slink back, having ineffectually attacked causality itself, to arguing that even if we granted the reasonableness of the God of the philosophers, how can we know that this God is at all related to the God of the Bible? When this occurs, I know that their gig is up.

    As to the second argument, I think that Feser would simply suggest that Rosenberg's purported use of 'conceive' was as loose as Hume's when he proposed his critique of causality, and I would humbly agree.

    ReplyDelete
  117. d_b,
    >Rosenberg's purported use of 'conceive' was as loose as Hume's

    Thanks, I didn't mean to imply R actually used that argument, only that I could see him doing so. I'll have to think about Hume. My purpose was to query how a Thomist would respond to anyone (philosopher or scientist) who argued, as a metaphysical position, that a causal series ordered per se indeed had a first uncaused member ... and that first member, or first principle, was what everybody knew as the Initial Uncaused Immaterial Natural Unified Force of the whole world.

    To make this possible metaphysical counter-position even clearer, what is the correct or best Thomist metaphysical position to take or advance if scientists succeed one day in unifying all known natural forces into one existent unified force (as Einstein and others have tried to do)?

    Is such a metaphysical postulate, as the Initial Uncaused Immaterial Natural Unified Force of the whole world, "unintelligible" and, therefore, irrational?

    ReplyDelete
  118. @James Chastek,
    "The second way starts by taking a series of efficient causes as a given. It does not say that there is an efficient case for everything, including things that happen by chance."

    I think that the logic of the second way requires that the chain run unbroken all the way back to God. If I am allowed to say "oh that, that was just radioactive decay", then someone could object that the chain of efficient causes did not terminate with something that everyone could agree was God, but with a vast set of accidents, independent and random, which even now continue to form the world.

    @OFloinn,
    "It is only that there is no cause as to why that particular person was struck by that particular hammer at that particular time."

    The nature of the particular event is not important. It can be even more trivial than a murder by hammer. The issue is that if such exceptions are permitted then it is not self evident that there is anything special about the end of a finite causal chain. It is not self evident that the world is intelligible at all. Under such circumstances why should I care about or believe the principle of causality?

    ReplyDelete
  119. And to be clear, I have (this time) intentionally dropped Aristotle out of the expression. I am not a scholar in Aristotle-Thomistic philosophies, but I my sense is that Aristotle might not have a problem with my formulation. I am ready to be schooled on that, of course, as I am on Thomistic matters. That is why I am here. Peace.

    ReplyDelete
  120. It is not self evident that the world is intelligible at all.

    Somehow, people who start down that path often wind up at this point, throwing out natural science on the ash heap. So much for logic and reason. But... "What do we want?" X! "When do we want it?" NOW! When all thought is geared around what one wants to be true -- Nietzsche is so dramatic and philosophical egoism so wonderfully self-satisfying that any hard work of the intellect is abandoned and, if something is not "self-evident" then why bother?

    ReplyDelete
  121. To add one last clarity, I also intentionally left the word "Intelligent" and the word "Unintelligent" OUT of the full expression: Initial Uncaused Immaterial Natural Unified Force of the whole world. I am not trying to be clever here, because the issue of "intelligence" is part of the actual formation of my question; that is, what is and is not intelligible.

    ReplyDelete
  122. c emerson, to say that the world is intelligible is not to say that it is itself intelligent but rather that it is able to be understood. Or, to purloin a phrase of Oakeshott's, the universe is not so much intelligence as an exhibition of intelligence. That is why A-T argues that intelligibility is a part of the philosophy of nature that is presupposed by the sciences and that materialism understood as a philosophy of nature is incoherent.

    As to your first comment, how is that position different to the God of the philosophers?

    ReplyDelete
  123. C Emerson: And isn't Feser [explaining Aquinas] grounding his /Aquinas' argument on an axiomatic premise that the human mind [indeed any mind] cannot reasonably conceive [or maintain or hold] that a causal series ordered per se can infinitely regress?

    It's only something that cannot be reasonably conceived insofar as the idea is not reasonable, i.e. is wrong. No more could someone respond to Pythagoras's Theorem by claiming that he could "conceive" of a right-triangle where the square of the sides did not sum to the square of the hypotenuse. As is sometimes noted, it's not even that an infinite per se causal series has to be impossible, but that it's impossible for such a thing to escape the need for a first cause. You could claim to be able to conceive an endless chain of e-mail that was never written by anyone, or an infinite number of moons reflecting sunlight when there is no sun, but you would be claiming to conceive the impossible.

    ReplyDelete
  124. James Chastek: One thing I find baffling about all invocations of QM in this context is that people seem to think that, until QM, no one had ever realized that some things happen by chance.

    I don't quite think it's that; if Laplace's demon knew all the different causal chains at work, he could predict precisely when and how they would coincide. But if one has decided that everything reduces to matter, and that matter reduces to physics, then if physics cannot describe some cause (aka predict something), then one is going to conclude that there is no such cause.

    Of course, the better conclusion would be that one was mistaken in deciding that reality was nothing more than physics; but some folks, when they find they have dug themselves into a hole, just keep digging.

    ReplyDelete
  125. Anonymous: Serious question guys: What on earth makes you think the bizarre absurdity depicted in the Bible should be identified with the ground of being?

    Funny, that doesn't sound like a very serious question.

    Anonymous: I don't intend to buy Feser's book so don't waste your time. It's ok if you didn't understand his argument and don't feel competent to discuss it. But if you actually understood what he was saying I encourage you to present his views here.

    And I encourage people not to. Repeating the same things over and over again, typically in less polished form, is the bane of website commenting. Anyone who can't be bothered to get Feser's book — or even borrow it from the library — is clearly not prepared to put much effort into the matter. (It's not a big book, it's easy to read, and it covers the necessary basics.) It's silly to waste time trying to explain something to someone who isn't really interested in the answers.

    Now, if someone has read the book (or the many, many articles posted on this site), and has questions about some certain point that can be clarified, then that makes for a constructive use of the comments. It's far more productive, and produces a higher quality of discussion, and I would hope that that's the sort of thing we all would like to encourage around here.

    ReplyDelete
  126. C Emerson: and that first member, or first principle, was what everybody knew as the Initial Uncaused Immaterial Natural Unified Force of the whole world.

    When Aquinas says, "this everyone calls God", it's more of a definition. The argument shows that there's some Primary Cause or First Mover, something that whatever it is is responsible for the whole universe and everything in it. I certainly would call that a god, indeed, since it's at the very top, the God. So would most other people. Whether that Being is YHWH or whether it's "the laws of physics" or what have you is a different question. And that's why Aquinas spends the next 49 chapters investigating what this First Cause is like. Turns out it doesn't sound anything like "self-creating gravity".

    Is such a metaphysical postulate, as the Initial Uncaused Immaterial Natural Unified Force of the whole world, "unintelligible" and, therefore, irrational?

    Well, it's irrational insofar as a Unified Force (in any scientific sense) doesn't have the properties that the Prime Mover has to have. You could call it that if you really wanted to, but of course given how those terms have been used historically, it would be confusing. Indeed, one the reasons why we automatically identify "God" or the "Prime Mover" as referring to God in the Biblical sense and not as some sort of pantheistic force of nature is simply because these arguments have been so successful in showing that "God" is like one and not like the other. When someone objects that Aquinas is smuggling in preconceptions about what God is into the argument, it is the objector who really has the preconceptions, because these notions have so thoroughly percolated into the fabric of (Western) culture.

    ReplyDelete
  127. Anonymous, but we don't know which one, said: (This is not the anon you were responding to by the way)

    Tip to anonymous posters: you can be just as anonymous by choosing "Name/URL" and making up a random* pseudonym. I don't even care if you make up new names for each different thread, but it really does help to be able to identify a speaker through the course of a single conversation! Thanks.



    [*Randomness not to be confused with lack of causality. Please follow the laws of Being at all times. No refund without proof of purpose.]

    ReplyDelete
  128. d_b,
    > how is that position different to the God of the philosophers?

    That is exactly my question to the Thomists.

    Feser in Aquinas [p.66], quoting Aquinas in his ST with respect to the concluding part of the argument of the First Way, says: "... if there were no first mover, there would be no other movers .... It follows that 'it is necessary to arrive at a first mover, put in motion by no other; and this everyone understands to be God' (ST I.2.3)."

    Why did Thomas Aquinas use the word "God" here rather than a more general expression, say one similar to my expression, Initial Uncaused Immaterial Natural Unified Force?

    [I realize Aquinad would not have been familiar with the notion of "unified force" but he could have found other appropriate terms for this without selecting the term "God"]. My point is, what is the Thomist argument that it is God rather than an eternally existent natural force that is the necessary prime mover? Thanks.

    ReplyDelete
  129. C emerson, see Mr Green's response immediately above. My question was not suggesting that there is no difference so much as an invitation for you to describe what you imagined the differences were.

    ReplyDelete
  130. Yes, I just saw Mr Green's posts after I posted.

    > It's only something that cannot be reasonably conceived insofar as the idea is not reasonable, i.e. is wrong. No more could someone respond to Pythagoras's Theorem by claiming that he could "conceive" of a right-triangle where the square of the sides did not sum to the square of the hypotenuse.

    I appreciate the clarity of this response, do not necessarily agree that conceiving of an infinite causal series ordered per se is the same (or analogous) to the Pythagorean Theorem example, but accept that that is what Aquinas had in mind. Thanks to you both for the explanations and the effort.

    > And that's why Aquinas spends the next 49 chapters investigating what this First Cause is like. Turns out it doesn't sound anything like "self-creating gravity".

    As to the definitional use of the word "God", I accept that as well. That is in fact part of my point in this whole discussion of Craig versus Rosenberg. If a non-Thomist were to accept in general the argument of the First Way, such a non-Thomist is not necessarily committed to the next 49 chapters of analysis as to what this prime mover is ... which is not to say that Aquinas is necessarily wrong about that analysis, only that it is a separate, or st least an extended issue. Thanks again.

    ReplyDelete
  131. This anon... Doesn't he look like a certain Potty we use to have around here, he uses all the same arguments and knows a lot of stuff about past comments in blog.

    And has a tremendous fixation about God and sex! Just like potty did hahahha.

    Seriously, I think we might be discussing with the guy all along XD.

    ReplyDelete
  132. @d_b

    Incidentally, I did not mean to imply, with my use of the word "intelligent" or "unintelligent" that I mean to describe an attribute of the natural world. I probably should have included the word "being" as in "intelligent being". I obviously don't mean to imply here an intelligent being status to my use of Initial Uncaused Immaterial Natural Unified Force. Instead, as brought out by Mr Green, the question of WHAT the prime mover would have to be, would subsume the question of intelligent being. Thanks for this exchange.

    ReplyDelete
  133. Eduardo, it is hard to say one way or another because they all sound more or less the same, which is not surprising since they are generally parroting the tropes of a Dawkins or Hitchens.

    ReplyDelete
  134. Yes, but the similarities are really unique, esoecially the usual bitching about Feser's book being too long hahhhaa. Now we have this example about sex, which was potty's favorite. Maybe it is JUST dawkins by somebody else, who knows...

    ReplyDelete
  135. Sex always comes up, Eduardo. Hitchens was always raising the Church's teaching on contraception and masturbation. I honestly think that this is the substantive motive force of atheism in adolescent males, particularly.

    ReplyDelete
  136. "95% of the National Science Academy are atheists" Rosenberg.

    What follows by that logic? That truth is determined by popular vote. Or a poor metaphysical foundation for scientific inquiry?

    ReplyDelete
  137. @Mike's Creative Writing Blog:

    "95% of the National Science Academy are atheists Rosenberg.

    What follows by that logic? That truth is determined by popular vote. Or a poor metaphysical foundation for scientific inquiry?"

    It is worse than that. For what special competence have scientists in gauging the merits of *philosophical* arguments? None. It has about the same value has saying that 95% of a would-be guild of plumbers were all atheists. Interesting factoid, but the correct response is so what?

    ReplyDelete
  138. (I realize I come to this quite late.)

    Although Rosenberg behaved ungraciously in his opening statements and I cannot condone his rancor, I largely agree with the content of his personal remarks on Craig. Craig has always struck me as full of a smarmy smugness, interested in winning debates rather than seeking the truth; I’m discouraged to see him used as a club to bash those who may be uncomfortable or unskilled debaters. After all, the question of God’s existence won’t be decided after a few hours of nontechnical philosophical debate anyway. It feels a bit beside the point to say “Ha ha, so-and-so won’t debate Craig, he must lack courage in his convictions!”

    I’d be a terrible debater — a nervous speaker, poor at thinking on my feet, prone to debilitating dry mouth and confused rambling. But that doesn’t mean that I’m afraid to defend my beliefs (such as they are) where challenges to them are appropriate.

    ReplyDelete
  139. Quick question:

    How can an infinite series require a first cause, if "infinite" just means "no end"? A first cause would be an end to the infinite chain, and thus it would not be infinite at all.

    Or am I missing something?

    ReplyDelete
  140. >How can an infinite series require a first cause, if "infinite" just means "no end"? A first cause would be an end to the infinite chain, and thus it would not be infinite at all.

    Come on dguller you should know better. Are we talking about an essential series or accidental one?

    Can you explain the motion of a caboose by hooking up an infinite series of box cars in front of it?

    At some point you need a locomotive.

    OTOH if you had an infinite series of boxcars pulling a caboose you would still need a transcendent outside explanation to account for it.

    Why is this hard? This is TLS 101.

    Cheers.

    ReplyDelete
  141. A start with no end perhaps? Like the set of numbers [0,infinity). It has a definite boundary, which is zero. But there are still an infinite amount of numbers in the set. (I'm not a math guy though.)

    ReplyDelete
  142. Ben:

    Can you explain the motion of a caboose by hooking up an infinite series of box cars in front of it?

    But then the infinity is going in one direction only. Take a straight line. It can be infinite in length, but not width. So, you have to specify what limit you mean when you say “no limit”.

    With regards to a per se series of causes, the infinity could not be bidirectional. It could only be unidirectional, going from the first cause onward through an unlimited series of subsequent per se causes. It cannot be bidirectional, because there would have to be no limit stretching backwards or forwards, which necessarily eliminates the possibility of a first cause. After all, a first cause would be a limit to the per se series moving backwards in metaphysical priority, and thus the series would not be infinite in that direction, only really, really large.

    ReplyDelete
  143. This comment has been removed by the author.

    ReplyDelete
  144. Let me try to expand on Ben's example. Say we are floating in space, and there is a seemingly infinite chain of boxcars moving in front of our eyes. No start or end in sight. Now, we pick one of the cars, Car 1, and find that it is accelerating at 10 meters per second squared. We ask for an explanation as to why Car 1 is moving at 10 meters per second squared. One could say that the acceleration is due to car behind or in front of Car 1, which are Car 2 and Car 0. But what is accelerating Car 0? We could say that it is due to Car -1 and/or Car 1. And so on to infinite. However, the problem with that "so on and so forth" explanation is that it fails to explain why the acceleration of 10 meters per second squared exist at all, when none of the members of the line have the mechanism to produce it in the first place. The boxcars can "transmit" acceleration, but they do not have an engine, or any other means to accelerate on their own.

    ReplyDelete
  145. Grodrigues:

    First is not in the sense of that which comes before the last, the first to last, etc., it is first in ontological priority.

    I understand that.

    

I suppose you *could* reason like this: even if such a per se chain were to exist, it itself would have to derive the causal efficacy from somewhere since it is purely instrumental, that is, we can still ask the same question for the whole chain. Apply transfinite induction to prove that the chain must stabilize somewhere.

    But that first cause would be the terminus for the causal sequence, whether the sequence was temporal or ontological. And if the sequence has a terminus, then it cannot be unlimited, and thus it cannot be infinite. Like I said, I have no problem with the possibility of an infinite per se causal series if it starts at the first cause, and then proceeds to infinity. But if the infinite per se causal series is supposed to extend backwards, either temporally or ontologically, then it cannot – by definition – stop at a first cause, because it cannot stop at all, because it is unlimited in that direction.

    ReplyDelete
  146. @dguller:

    "How can an infinite series require a first cause, if "infinite" just means "no end"? A first cause would be an end to the infinite chain, and thus it would not be infinite at all."

    First is not in the sense of that which comes before the last, the first to last, etc., it is first in ontological priority.

    I suppose you *could* reason like this: suppose such a chain does exist. By induction, no element is in act in the relevant sense. This entails that the whole sequence is causally inert, so it too must have a cause. Such a cause becomes the sequence's real first member. If you want to go up the transfinite hierarchy or introduce causal loops, no problems either.

    edit: previous post deleted and argument in the second paragraph corrected.

    ReplyDelete
  147. Anonymous:

    Say we are floating in space, and there is a seemingly infinite chain of boxcars moving in front of our eyes.

    I’ll stop you right here. I’m not talking about a seemingly infinite series, but about an actually infinite series. I completely agree that a series that seems to be infinite could actually be limited, but I disagree that a series that is defined to be infinite in a particular aspect can be limited in that same aspect. So, if a line is infinite in terms of the aspect of length, then if it stops at some point along its length, then it cannot be limited in terms of length.

    ReplyDelete
  148. "I’ll stop you right here. I’m not talking about a seemingly infinite series, but about an actually infinite series. I completely agree that a series that seems to be infinite could actually be limited, but I disagree that a series that is defined to be infinite in a particular aspect can be limited in that same aspect. So, if a line is infinite in terms of the aspect of length, then if it stops at some point along its length, then it cannot be limited in terms of length."

    Yeah, that was a mistake on my part. However, let us assume that the observers know for a fact that there are an infinite number of boxcars. If you go in the same direction as the boxcars are accelerating, you will never reach an end. Also, if you travel in the opposite direction, you will never reach an end. Now just plug in the rest of the scenario.

    ReplyDelete
  149. This comment has been removed by the author.

    ReplyDelete
  150. >But then the infinity is going in one direction only. Take a straight line. It can be infinite in length, but not width. So, you have to specify what limit you mean when you say “no limit”.


    So what? The point is there can be no infinite essential series only at best an infinite accidental series.


    >With regards to a per se series of causes, the infinity could not be bidirectional. It could only be unidirectional, going from the first cause onward through an unlimited series of subsequent per se causes. It cannot be bidirectional, because there would have to be no limit stretching backwards or forwards, which necessarily eliminates the possibility of a first cause. After all, a first cause would be a limit to the per se series moving backwards in metaphysical priority, and thus the series would not be infinite in that direction, only really, really large.

    Who cares? How does that overthrow the accidental causal vs the essential causal series distiction? It doesn't matter to me nor is it relavent to speculate wuther a caboose being pulled by an infinite series of boxcars has the power to stop & back up or not.

    That's not the point of the argument so those particulars aren't interesting or relavent IMHO.

    ReplyDelete
  151. Ben:

    So what? The point is there can be no infinite essential series only at best an infinite accidental series.

    The point is that if a bidirectional infinite per se causal series was actually possible, then there would be no need for a first cause, by definition. On the other hand, if a bidirectional infinite per se causal series is impossible, then there would have to be a limit somewhere, and that limit would be the first cause, which is fine. My only objection was to the idea that even if there was a bidirectional infinite per se causal series, then you would still need a first cause. That’s just not true, by definition.

    ReplyDelete
  152. Grodrigues:

    By induction, no element is in act in the relevant sense.

    What is the “relevant sense”? There is act and there is potency. If the members of the series are actual, then they are in act, and thus causally efficacious. Maybe you mean that they are composite entia with an admixture of act and potency, rather than being metaphysically simple and pure actuality?

    This entails that the whole sequence is causally inert, so it too must have a cause. Such a cause becomes the sequence's real first member. If you want to go up the transfinite hierarchy or introduce causal loops, no problems either.

    But then it is not an infinite per se causal series. It is a finite per se causal series, because its limit is the first cause. That’s all I’m saying. It is impossible to have an infinite per se causal series and a first cause, at least in the relevant direction, whether temporal or ontological.

    ReplyDelete
  153. and this everyone calls God should be read as and this everyone calls God, details to follow.

    unified force
    A force is a relationship between two existing things. Since X cannot cause diddly squat if it does not yet exist, an imaginary force cannot be identified with the first actualizer. Gravity, for example, is a term we use to say that one material body attracts another. As near as we can tell today, that is because the existence of a material body causes a warping in the field of Ricci tensors that then results in motion of other bodies nearby.
    + + +
    a non-Thomist is not necessarily committed to the next 49 chapters of analysis

    Example: Primary Actualizer must be purely Actual, uncompounded with Potency, because if it were in potency to anything, it could be moved by something more actual than itself and hence would not be primary.
    Example: Given this, there cannot be two such beings because, to be two, there must be some distinction between them, and one would possess X and the other not. But then the other would be in potency to X and therefore not a Primary Actualizer.
    And so on. I'm not sure why this toppling of logical dominoes would be "uncommitted to," given the initial theorem.
    + + +
    I’d be a terrible debater — a nervous speaker, poor at thinking on my feet, prone to debilitating dry mouth and confused rambling. But that doesn’t mean that I’m afraid to defend my beliefs (such as they are) where challenges to them are appropriate.

    No, it means you would not be able to defend them, period. You would have to hire someone skilled in logical thought.

    ReplyDelete
  154. Perhaps instead of saying "first" or "ontological first," we should just say "sustaining." Unless "ontological first" really does mean "sustaining" in A-T terminology, kind of like how "motion" means "change."

    ReplyDelete
  155. Anonymous:

    However, let us assume that the observers know for a fact that there are an infinite number of boxcars. If you go in the same direction as the boxcars are accelerating, you will never reach an end. Also, if you travel in the opposite direction, you will never reach an end. Now just plug in the rest of the scenario.

    But then it doesn’t work. You can just say that each boxcar gets its causal efficacy from the one before it, and since it goes on forever, there is no beginning or ending to the causal chain. And that fatally compromises the need for a first cause, at any level of analysis. You can look at it temporally, ontologically, horizontally, vertically, or whatever. If you admit the possibility of an infinite series, especially going “backwards” (in whatever relevant sense), then you have forfeited the need for a first cause at all.

    Fortunately, there are good reasons for doubting that such infinite series are possible at all, and so it’s not a fatal objection to the First Way, for example.

    ReplyDelete
  156. Anonymous:

    Perhaps instead of saying "first" or "ontological first," we should just say "sustaining." Unless "ontological first" really does mean "sustaining" in A-T terminology, kind of like how "motion" means "change."

    Perhaps a better way of putting it would be making a distinction between horizontal causality and vertical causality. Horizontal causality would be the temporal kind of causality, and vertical causality would be the kind of sustaining causality that you are referencing.

    If you say that a per se causal series, whether horizontal or vertical, can be infinite in the “prior” direction, then you have forfeited the need for a first cause, because if there is a first cause, then the series cannot be infinite. Perhaps a horizontal per se causal series can be infinite, but a vertical per se causal series cannot be infinite, and must have a limit or terminus in the first cause.

    ReplyDelete
  157. But that first cause would be the terminus for the causal sequence

    Why? You already said you understand that "first" does not mean "before the next." But here you seem to think that the "first" cause is one of the members of the series, and therefore "just another boxcar" or "just another forwarder."

    Here is an infinite series:
    {X|X(n) = n*X(n-1)}
    That series has a first cause, namely me, who typed it at 2:39 EDT 6Feb2013. Yet it is itself unending and unbeginning. The solution? I am not myself a number in the series, any more than Sharon Kam is a note in Mozart's Clarinet Concerto in A.

    ReplyDelete
  158. >The point is that if a bidirectional infinite per se causal series was actually possible, then there would be no need for a first cause, by definition.

    Nonsense how would you then explain it's initial motion? You are making djindra's classic mistake of assuming the example is the argument.

    >But then it is not an infinite per se causal series. It is a finite per se causal series, because its limit is the first cause.

    Your equivocating your terms and senses. The series is infinite in the sense there are not a finite number of box cars. In either direction there is no end to the box cars and you will never come to the end and it's initial motion can only be accounted for a cause outside the series.

    I don't see how you would conceive of the outside cause as "finite" since it has to keep an infinite series of cars in motion?


    >That’s all I’m saying. It is impossible to have an infinite per se causal series and a first cause, at least in the relevant direction, whether temporal or ontological.

    Yes this reminds me of St. Bonaventure argument as to why you can have a past eternal universe who modern manifestation is part of the KCA. But what does that have to do with anything? The point of the argument is even if you have an infinite accident series you need an outside cause to account for it's being.


    >On the other hand, if a bidirectional infinite per se causal series is impossible, then there would have to be a limit somewhere, and that limit would be the first cause, which is fine. My only objection was to the idea that even if there was a bidirectional infinite per se causal series, then you would still need a first cause. That’s just not true, by definition.

    By definition that is absurd. One could conceive of an infinite number of locomotives moving on an infinite track(in both directions) but not an infinite number of box cars unless it's mover was on outside(in the transcendent sense) moving it.

    Of course even the locomotives aren't the cause of their own existence so top down you still need to account for their existence.

    OTOH if your argument is that you can't have an actual infinite series of box cars in either direction that is still not relevant to the argument since it would even more sharply argue for the need for an outside infinite metaphysical cause.

    Unless you can explain to me what "finite" cause can move something infinite.

    ReplyDelete
  159. "Perhaps a better way of putting it would be making a distinction between horizontal causality and vertical causality."

    Perhaps an even better way would be to distinguish between primary and secondary causes, as A-T in fact does. Even a bidirectionally infinite series in which each element is a secondary cause of the next still requires a primary cause in order to exist at all -- which is exactly the point everyone has been making in reply to your question.

    ReplyDelete
  160. You can just say that each boxcar gets its causal efficacy from the one before it, and since it goes on forever, there is no beginning or ending to the causal chain. And that fatally compromises the need for a first cause

    Try to follow this: a boxcar in and of itself does not possess the power to move another boxcar. It doesn't matter how many boxcars are in the sequence. An infinite number of zeros does not add up to 42.
    + + +

    ReplyDelete
  161. TOF:

    Why? You already said you understand that "first" does not mean "before the next." But here you seem to think that the "first" cause is one of the members of the series, and therefore "just another boxcar" or "just another forwarder."

    It doesn’t have to be part of the series, but it has to be actually there acting upon the members of the series, and that is because its activity is what causes the existence of the series at any point in the series. And prior to its activity, there is no series at all, which is why it is supposed to be first.

    Here is an infinite series: 
{X|X(n) = n*X(n-1)} 
That series has a first cause, namely me, who typed it at 2:39 EDT 6Feb2013. Yet it is itself unending and unbeginning. The solution? I am not myself a number in the series, any more than Sharon Kam is a note in Mozart's Clarinet Concerto in A.

    Why is it “unbeginning”? You are its beginning. Prior to you, it did not exist.

    ReplyDelete
  162. Scott:

    Perhaps an even better way would be to distinguish between primary and secondary causes, as A-T in fact does. Even a bidirectionally infinite series in which each element is a secondary cause of the next still requires a primary cause in order to exist at all -- which is exactly the point everyone has been making in reply to your question.

    Whatever you want to call it, the bottom line is that it is impossible to have an infinite per se series of primary causes. And that’s fine.

    ReplyDelete
  163. TOF:

    Try to follow this: a boxcar in and of itself does not possess the power to move another boxcar. It doesn't matter how many boxcars are in the sequence. An infinite number of zeros does not add up to 42.

    Why does the boxcar have to have intrinsic power? The boxcar is pulling the boxcar attached to it. How is it doing this? Because the boxcar before it is pulling it. How is it doing this? Because the boxcar before it is pulling it. If this series is allowed to go on forever, then there is no justification for any first train. After all, it is already granted that there is no end to the series, and thus there cannot be a first train pulling all the rest. The only problem is if you grant for the sake of argument that there can be an infinite series of boxcars, each pulling the other, extending in the direction of the pulling forever. If you make that move, then by definition, there cannot be a first puller. So, why make that move? Just argue that such an infinite series in the pulling direction is impossible, and the argument is strengthened.

    ReplyDelete
  164. Why is {X|X(n) = n*X(n-1)} “unbeginning”? You are its beginning. Prior to you, it did not exist.

    Let's reword the analogy. You contend:
    Why is the World “unbeginning”? God is its beginning. Prior to God, the World did not exist.

    But now you are reasoning in a circle. You are assuming God rather than proving God. To do this, you must use "beginning" in an equivocal manner. I brought the series into being, but I am not a number in the series. If you restrict yourself to being "inside" the series, then you are a wild-eyed god-thumper to postulate the "TOF hypothesis."

    ReplyDelete
  165. >Why is it “unbeginning”? You are its beginning. Prior to you, it did not exist.

    There you go with the equivocating.

    As a mathematical formula it has no formal mathematical beginning here like let us say 1+1=2 with 1 being the beginning of this addition series.

    Why is this hard for you?

    ReplyDelete
  166. "Whatever you want to call it . . . "

    It's not a question of what one or the other of us wants to call it. Your question was about first (or "primary") causes.

    Your own distinction between horizontal and vertical causality conveys part of the point well, but it doesn't convey what A-T actually means by a first cause and therefore leaves out the fact that the "vertical" cause is ontologically prior to all the "horizontal" ones. That distinction thus leaves your own question unanswered.

    " . . . the bottom line is that it is impossible to have an infinite per se series of primary causes. And that’s fine."

    I assume (and hope) you're expressing your agreement with A-T here. Aquinas himself acknowledged that there might be bidirectionally infinite accidental series of secondary causes (or at least he didn't see any way to prove otherwise); it was the possibility of a bidirectionally infinite per se series that he denied. (And of course either sort of series requires a "primary cause" in order to exist at all.)

    ReplyDelete
  167. Everyone:

    There is no disagreement here. If we all agree that it is impossible for a per se causal series -- whether horizontal or vertical, primary or secondary, or whatever – to stretch to infinity bidirectionally, then we are in agreement. I only brought up the topic, because TOF (at February 5, 2013 at 11:02 AM) and Mr. Green (at February 5, 2013 at 10:11 PM) made comments that made it seem that they endorsed the possibility of an actual per se infinite series. My problem with this is that once you have accepted the possibility of an actual per se infinite causal series – particularity of a vertical or primary kind -- then you have compromised the argument for a first cause. As Feser writes: “there cannot be an infinite number of causes that are per se required for a certain effect” (Aquinas, p. 71) and “To suggest that such a series might regress infinitely, without a first member, is therefore simply unintelligible” (Ibid., p. 72).

    ReplyDelete
  168. I doubt the possibility of an actual infinite series, period, on grounds of physics. But the very point is that even if the world is eternal, and therefore a chain of physical causes may itself be eternal, an essentially-ordered series must have a primary cause regardless. You have gone three sides around the barn and are confusing primary causation as if it were another "direction" of the same sort of thing as secondary or instrumental causation.

    ReplyDelete
  169. @TOF

    I doubt the possibility of an actual infinite series, period, on grounds of physics.

    Which part of physics denies the possibility of an actual infinite series?

    ReplyDelete
  170. Maybe... cosmology and thermodynamics?

    ReplyDelete
  171. Dguller: I only brought up the topic, because TOF (at February 5, 2013 at 11:02 AM) and Mr. Green (at February 5, 2013 at 10:11 PM) made comments that made it seem that they endorsed the possibility of an actual per se infinite series. My problem with this is that once you have accepted the possibility of an actual per se infinite causal series – particularity of a vertical or primary kind -- then you have compromised the argument for a first cause.

    Actually, I agree. There might be a set of infinite causes where the primary is "vertically" on top and there are infinite members "horizontally", but in the end that means that the infinite chain is not what's pertinent to the argument. If the "top" cause is the first (causally or ontologically), then we actually have only a two-member series in the relevant respect: top and bottom. So from now on, I shall stick to what Thomas said in the first place: it is impossible to proceed back to infinity in a causal chain that is per se (in the relevant way!).

    (Incidentally, it may be possible to have a series that is infinite in the "middle": e.g. consider the closed interval [0...1]. It's interesting to contemplate how that might work causally, but it still doesn't affect the argument because the attempted objection relies on not being able to trace back to a first member; whereas in this case we have clearly identifiable endpoints, namely zero and one.)

    ReplyDelete
  172. One last comment, even though things seem to be resolved. I don't think there needs to be an "end" in the train chain for there to be an "accelerator." It could be at any point within the chain, because it could pull and push at the same time. (Picture a man who is pushing a box while he has another box chained to his leg.) Also, the "accelerator" need not be part of the infinite train chain at all, but could be "dragging" the train chain along from the outside. If the accelerator ceased the dragging, the infinite train chain would stop accelerating, and would start moving at constant velocity.

    ReplyDelete
  173. @TheOFloinn:

    “No, it means you would not be able to defend them, period. You would have to hire someone skilled in logical thought.“

    I have admitted to nothing more consequential than a nervousness caused by addressing crowds, which inhibits my ability to persuade relative to someone more practiced and confident. I don’t see how this implies that I am totally incapable of logical thought: I am happy and willing to engage others in writing or among a more modestly-sized group of people, and I feel such settings are more likely to get nearer to the truth anyway.

    To put it another way — by the nature of the debate format, factors unrelated to the truth of the matter can be a huge part of determining the “winner”. For someone whose strengths lie elsewhere, debating someone of Craig’s practiced confidence offers a losing proposition. He could argue that the moon landing was a hoax perpetrated by Martians and I’d give him good odds against most people.

    So reluctance to debate might be good sense rather than rank cowardice.

    ReplyDelete
  174. @Mr. Green:

    "There might be a set of infinite causes where the primary is "vertically" on top and there are infinite members "horizontally", but in the end that means that the infinite chain is not what's pertinent to the argument. If the "top" cause is the first (causally or ontologically), then we actually have only a two-member series in the relevant respect: top and bottom. So from now on, I shall stick to what Thomas said in the first place: it is impossible to proceed back to infinity in a causal chain that is per se (in the relevant way!)."

    That is my take also.

    Even if for a moment we granted the existence of per se infinite causal chains, the argument would *still* not be defeated because the heart of the argument is to establish that there is an absolute first member in the ontological sense that actualizes the whole chain.

    In this view, dguller's protests, at least as far as I can understand them, are just arguments for the finite character of the chain which as far as the argument goes, boils down to the same.

    ReplyDelete
  175. Ben said this: >OTOH if you had an infinite series of boxcars pulling a caboose you would still need a transcendent outside explanation to account for it.

    Let me support Ben on this one, and from what I am now reading, dguller and Scott, too.

    Aquinas (as explained by Prof Feser, not by me) draws a clear distinction between a causal series ordered per accidens (no additional adjectives) and a causal series ordered per se (no additional adjectives).

    The first can theoretically be infinite.

    The latter must be finite.

    This is covered in Feser's book, Aquinas [p.71], where Feser writes: "... it is in Aquinas's view at least theoretically possible for a causal series ordered per accidens to regress to infinity, and thus have no beginning point(ST I.46.2)."

    Feser then writes (quoting Aquinas): "By contrast, in 'efficient causes it is impossible to proceed to infinity per se - thus, there cannot be an infinite number of causes that are per se required for a certain effect...' (ST I,46.2)."

    Finally Feser writes [p.72; without any direct reference to a writing by Aquinas]: "To suggest that such a [per se] series might regress infinitely, without a first member, is therefore simply unintelligible."

    ... which is exactly why I asked for more explanation as to what distinguishes an infinite per accidens series from a finite per se series. I think that us what dguller is asking, but I don't want to put words in his mouth here.

    ... it is also why I asked for more explanation as to the exact basis for declaring that it is "simply unintelligible" to "suggest" [or, in my words, 'conceive'] a per se causal series as regressing infinitely (as opposed to a per accidens causal series).

    I received some well-articulated answers to my questions (see dover_beach and Mr Green and some others above), which I appreciated, and which I am puzzling over.

    And to extend this comment just a bit, Ben is saying (I think) that there is also another distinction between a finite causal series per se and an infinite (eternal) being (God).

    And grodrigues just said > heart of the argument is to establish that there is in absolute first member in the ontological sense that actualizes the whole chain
    [italics added by me].

    I agree with that statement, which is why I asked my second question about "unintelligibility" the way I did.

    Hope that helps -- btw, I don't necessarily agree with the answers, but I believe I now understand Feser's arguments / explanations on p.68-73 of his book. Peace.

    ReplyDelete
  176. Which part of physics denies the possibility of an actual infinite series?

    The part with the Big Bang at one end and the heat death at the other.

    Of course, being Science™, this cannot be known with the certainty of mathematics or even of metaphysics. That's why I said "doubt the possibility" rather than "deny the possibility."

    The tentativeness of Science™ is no reason to either disregard it or to imagine fantasy universes. But it is why it is not relied upon by Aquinas, et al. for metaphysical proofs.
    + + +

    Regarding number intervals. Suppose you were an intelligent real number living in the interval (0,1). This is your universe. As long as you insist that the only reality is inside your universe, you will never discover "empirical evidence" of 0. You can approach it arbitrarily closely, but you can never reach it.

    ReplyDelete
  177. "The part with the Big Bang at one end and the heat death at the other."

    Let's start with "now." On the left-hand side we'll add the time (in some suitable frame of reference) halfway between "now" and the Big Bang; then the point halfway between that point and the Big Bang; and so forth. on the right-hand side we'll add the time halfway between "now" and heat death . . . You get the idea.

    In short, I think you're right to doubt rather than deny that physics rules out actual infinite series. If time is infinitely divisible, then something along the lines of what I've just constructed (or rather, of course, a much more rigorous version thereof) appears to be a bidirectionally infinite series in real spacetime.

    ReplyDelete
  178. While I’m here, totally unrelated to anything:

    Scott, years ago I discovered your old website just as I began to wonder if the Objectivists I’d fallen in with really knew what they were talking about. You may have been the single biggest influence on my development through that time. Without your writings I suspect my mind would have remained closed off to competing points of view, at least for a good while longer — and whenever I step back, take a deep breath, and try to see something from my opponent’s point of view, I think of Blanshard and the rational temper. So, um, thanks.

    Small world.

    ReplyDelete
  179. So reluctance to debate might be good sense rather than rank cowardice.

    None of these explanations work for Dawkins, who is more than happy to debate other people (usually people he seems to think he can defeat decisively, though that's bitten him before; see Lennox, Williams). He and his followers have also in the past been very confident about his ability to defeat Christians in what amounts to a battle of intellect. Cowardice seems entirely appropriate as a suspected reason in Dawkins' case.

    As for Craig, accusing him of smugness would make more sense if he wasn't regularly debating some of the smuggest people on the planet. The same goes for confidence: Dawkins and company aren't exactly lacking it, certainly in what they display in conversation. Not every atheist is like this, sure, but the New Atheists largely are. Being confident and smug is practically a membership requirement, otherwise you're labeled an accommodationist and sent on your way.

    ReplyDelete
  180. Reighley,

    I think that the logic of the second way requires that the chain run unbroken all the way back to God. If I am allowed to say "oh that, that was just radioactive decay", then someone could object that the chain of efficient causes did not terminate with something that everyone could agree was God

    The second way proves that 1.) given a series of efficient causes, i.e. given we restrict ourselves to talking about nothing but efficient causes forming a chain, it follows that 2.) there is some first efficient cause in the series. In addition to this it claims that 3.) this first efficient cause is God. If you bring up some non-efficient cause, or some phenomenon where there is no efficient cause, you are simply talking about what the argument is restricting itself too.

    Mr. Green,

    if Laplace's demon knew all the different causal chains at work, he could predict precisely when and how they would coincide.

    I agree with this completely - but he would not have to say that there was a per se cause of the coincidence. Even if some coincidence or correlation between A and B is necessary, it does require a causal connection between them.

    ReplyDelete
  181. Reighley,

    I think that the logic of the second way requires that the chain run unbroken all the way back to God. If I am allowed to say "oh that, that was just radioactive decay", then someone could object that the chain of efficient causes did not terminate with something that everyone could agree was God

    The second way proves that 1.) given a series of efficient causes, i.e. given we restrict ourselves to talking about nothing but efficient causes forming a chain, it follows that 2.) there is some first efficient cause in the series. In addition to this it claims that 3.) this first efficient cause is God. If you bring up some non-efficient cause, or some phenomenon where there is no efficient cause, you are simply not talking about what the argument is restricting itself too.

    Mr. Green,

    if Laplace's demon knew all the different causal chains at work, he could predict precisely when and how they would coincide.

    I agree with this completely - but he would not have to say that there was a per se cause of the coincidence. Even if some coincidence or correlation between A and B is necessary, it does require a causal connection between them.

    ReplyDelete
  182. At one point Craig was very steeped in Aquinas. The first thing he ever said to me was stated through Stuart Hackett who was on the phone with me sometime around 1970 or later:

    "Bill asked if you've memorized Aquinas yet."

    ReplyDelete
  183. Scott. Congratulations, you have just rediscovered Zeno's paradox of Achilles and the tortoise. But this was more fully explored by Burley and other medieval Aristotelians in the doctrine of first and last moments.

    A set can be both open and bounded. And a set with an infinite number of members can have a measure of zero. (The irrationals comprise a set of measure zero in the reals, even though there are infinitely many irrationals.)
    + + +

    Even if some coincidence or correlation between A and B is necessary, it does require a causal connection between them.

    Exactly, Dr. C.
    Non-statisticians often misunderstand the expression "correlation is not causation" by taking it to mean that correlation is some weaker form of causation, say one where multiple causes are muddied together and must be discerned "statistically" rather then "mathematically."

    But it really means just what it says: correlation is not causation, full stop. It's not even the same kind of thing. Generally, a good correlation can be due to one of four things:
    1. A causes B
    2. B causes A
    3. Z causes both A and B
    4. Coincidence.

    I can use a correlation to predict B, given A and the regression model, even in case 3. (I could also do it in case 4, but that is risky in the extreme, like predicting stock markets from dress hemlengths.)

    The example I gave earlier in the Threatened Anecdote of high pressure in a vessel (A) being correlated with low yields (B) is a case in point. Both resulted from a contaminant in a raw (Z). Given the causal relationships Z→A and Z→B, you can predict B from A. But they are each at the "effect" ends of causal arrows.

    ReplyDelete
  184. @TheOFloinn:

    "The irrationals comprise a set of measure zero in the reals, even though there are infinitely many irrationals."

    Typo? Surely you mean the set of *rationals*.

    ReplyDelete
  185. "Scott, years ago I discovered your old website just as I began to wonder if the Objectivists I’d fallen in with really knew what they were talking about. You may have been the single biggest influence on my development through that time. Without your writings I suspect my mind would have remained closed off to competing points of view, at least for a good while longer — and whenever I step back, take a deep breath, and try to see something from my opponent’s point of view, I think of Blanshard and the rational temper. So, um, thanks."

    Thank you. That's very nice to know. Yeah, small world, this Internet.

    ReplyDelete
  186. "Scott. Congratulations, you have just rediscovered Zeno's paradox of Achilles and the tortoise. But this was more fully explored by Burley and other medieval Aristotelians in the doctrine of first and last moments.

    A set can be both open and bounded. And a set with an infinite number of members can have a measure of zero. (The irrationals comprise a set of measure zero in the reals, even though there are infinitely many irrationals.)"

    Thanks, I'm familiar with Zeno's paradoxes and I have a graduate degree in mathematics. I was simply pointing out a possible counterexample to your suggestion that the Big Bang and heat death rule out the existence of infinite physical series, by actually giving a brief description of how to construct a (bidirectionally) infinite series of physically real moments in time.

    ReplyDelete
  187. And yes, grodrigues is surely right that you meant to say the rationals comprise a set of measure zero in the reals.

    ReplyDelete
  188. @Scott

    I've heard this type of argument before. I think the first time was during a debate between Quentin Smith and William Craig.

    So here is my question... Isn't the assumption of infinite divisibility itself begging the question?Let me a elaborate, because begging the question might not be the correct term though.

    Let's say we have a time internal of 1 second. That second, both you and me know is finite. Because by the time I typed the word because that temporal internal began and ended as attested by my MacBook Pro clock.

    Now taking that second that we've established is finite we decide to divide it into two equal segments. Then we take those segments and divide them as well into to equal parts and on the story goes ad infinitum. I assume I am dividing 1 second an infinite number of times, so we are left obviously with an infinite series, correct?

    If so, then how exactly have we jumped from the observed fact that 1 second is finite to the assumed claim that it's an actual infinite series.

    To me it seems that the very assumption of dividing the second by infinity is itself the conclusion and is thus circular.

    I am a bit confused by this paradox and perhaps I might be wrong. Do you have any thoughts on what I just examined?

    ReplyDelete
  189. "Isn't the assumption of infinite divisibility itself begging the question?"

    It would be if I had made it. But you may notice that I expressly said "if time is infinitely divisible" and offered my proposed counterexample only provisionally. You've correctly singled out the point that has to be questioned in order to deny that the counterexample is genuine; it's entirely possible that time is quantized, in which case the counterexample could fail.

    ReplyDelete
  190. "Let's say we have a time internal of 1 second. That second, both you and me know is finite."

    Careful here. The time may be finite in length or extension but still contain an infinite number of points. The question is whether time (or spacetime) is physically continuous or discrete.

    My point, rephrased, is that it's possible to construct a bidirectionally infinite series on the open interval from 0 to 1 (that is, not including either endpoint), that goes . . . 1/16, 12/8, 1/4 1/2, 3/4, 7/8. 15/16 . . .

    In order to know that we can't do something similar with the "open interval" betwnn the Big Bang and heat death, we'd have to know that there's a limit to the "fine-grainedness" of spacetime -- that there's a minimum "size" for an interval of time and we can't physically get all the way down to mathematical points.

    That's why I said TheOFloinn was right to regard his own statement as speculative rather than definitively established.

    ReplyDelete
  191. Yikes.

    For 12/8 read 1/8, and for betwnn read between.

    ReplyDelete
  192. @James Chastek
    "The second way proves that 1.) given a series of efficient causes, i.e. given we restrict ourselves to talking about nothing but efficient causes forming a chain, it follows that 2.) there is some first efficient cause in the series. In addition to this it claims that 3.) this first efficient cause is God."

    I've just gone over some bits of the Summa part 1. Here are my thoughts:

    (1) Thomas's argument in Question 2 Article 3 for the existence of a first efficient cause does not treat uniqueness at all. So there is no particular reason given in that place to agree with Thomas that God is what he is talking about.

    (2) In Question 44 Article 1 he asserts several times that everything must have an efficient cause. His argument is that if the cause does not exist then the effect cannot exist. I take this to be as good a definition of efficient cause as any, so I am adopting it. This still does not ensure that all causal chains terminate at the same first element, but it does seem to settle the issue as to whether in Thomas' system everything must have an efficient cause. It also underscores the fact that what we mean when we talk about "exist" with reference to creatures is different than what we mean when talking about God.

    (3) In the part about the uniqueness of God, Question 11, I cannot see any obvious references to causation. He seems to hint at it when he says that because the whole world is one, it can have only one per se cause. I don't understand why he thinks this must be so.

    (4) at this point divine simplicity got involved again and I developed a headache.

    ReplyDelete
  193. @Scott

    that there's a minimum "size" for an interval of time and we can't physically get all the way down to mathematical points.

    What I don't understand is how is it the case that if a second is infinitely divisible, said second transforms from finite as previously established and infinite.

    ReplyDelete
  194. "What I don't understand is how is it the case that if a second is infinitely divisible, said second transforms from finite as previously established and infinite."

    There's no transformation. It's finite in one respect (duration) and infinite in another (the number of dimensionless mathematical points or "instants" it contains). The former is a measure of its extension in spacetime, the latter is a measure of the number of members it contains when regarded as a set.

    ReplyDelete
  195. Offtopic:

    Can heavy, radioactive elements be said to have the "tendency" or "inherent disposition" to decay into more stable forms?

    ReplyDelete
  196. "set of measure zero..."

    Dagnabbit!

    I was simply pointing out a possible counterexample to your suggestion that the Big Bang and heat death rule out the existence of infinite physical series, by actually giving a brief description of how to construct a (bidirectionally) infinite series of physically real moments in time.

    Yes, by using "infinite" in two different senses. The interval (0,1) has infinitely many points, but comes to an end at 0 and 1, so it is not infinite in length.

    But the obvious objection is that mathematical infinities are second abstractions from empirical reality and need correspond only usefully, not actually. (Example: nothing in the real world is actually distributed as a Gaussian.) So the properties of 0 and 1 vis a vis Big Bang and Heat Death may be an illuminating analogy, but not an actual correspondence. Otherwise, simply draw a line segment of any length. It contains an infinite number of hypothetical Euclidean points; yet it is not an example of a physically realized infinity.

    On this link, go to "articles" then scroll down to "Instantaneous change without instants" and click on full text.
    http://www.davidsoderberg.co.uk/

    A second useful article is "Cantor's transfinite numbers and traditional objections to actual infinity."
    http://www.thomist.org/jourl/2000/January/2000%20Jan%20A%20R.htm

    ReplyDelete